Basic Science and Structure of the Skin Flashcards

1
Q

1- This wound is in the first phase of healing. What is the best term to describe this initial phase?

A. Proliferative
B. Regenerative
C. Inflammatory
D. Vascularization
E. Remodeling

A

Correct choice: C. Inflammatory

Explanation: Wound healing can be described with 3 phases: inflammatory, proliferative, and remodeling. The other options represent different phases or aspects that are critical to wound healing, but inflammation is the initial stage.

How well did you know this?
1
Not at all
2
3
4
5
Perfectly
2
Q

2- Eccrine glands are found on the:

A. Labia majora
B. Labia minora
C. Glans
D. Prepuce
E. Vermilion

A

Correct choice: A. Labia majora
Explanation: Eccrine glands are present all over the body except on the vermilion of lips, glans, labia minora, nail beds, and inner prepuce.

How well did you know this?
1
Not at all
2
3
4
5
Perfectly
3
Q

3- This patient has metastatic colon cancer and biopsy was done to rule out cutaneous metastases. Pathology showed abundant granulation tissue. What is the best description of the cellular composition of this healing tissue?

A. macrophages, fibroblasts, endothelial cells
B. neutrophils, macrophages, eosinophils
C. keratinocytes, fibroblasts
D. eosinophils, macrophages
E. mast cells, endothelial cells

A

Correct choice: A. macrophages, fibroblasts, endothelial cells

Explanation: Granulation tissue is composed most correctly of macrophages, fibroblasts, endothelial cells. Eosinophils, keratinocytes, and mast cells are less prominent in the makeup of granulation tissue.

How well did you know this?
1
Not at all
2
3
4
5
Perfectly
4
Q

4- Which part of the matrix gives rise to the dorsal nail plate?

A. Distal matrix
B. Proximal matrix
C. Ventral matrix
D. Dorsal matrix
E. Lateral matrix

A

Correct choice: B. Proximal matrix

Explanation: The proximal matrix gives rise to the dorsal nail plate. The distal matrix gives rise to the ventral nail plate.

How well did you know this?
1
Not at all
2
3
4
5
Perfectly
5
Q

5- What is the major determinant of sodium chloride concentration of eccrine sweat that is secreted onto the surface of the skin?
A. Temperature
B. Acetylcholine concentration in nerve endings
C. Sweat rate
D. Dietary salt intake
E. Ph

A

Correct choice: C. Sweat rate

Explanation: The composition of human eccrine sweat includes inorganic ions (sodium chloride, bicarbonate, potassium), urea, lactate, ammonia, amino acids, proteases, and other proteins. The sweat rate is the major determinant of sodium chloride concentration of eccrine sweat that is secreted onto the surface of the skin. Eccrine sweat is secreted from eccrine glands via merocrine secretion. The substance secreted is initially isotonic but becomes hypotonic as NaCl reabsorption occurs in the eccrine duct. The sweat rate determines how much reabsorption occurs, and thus determines the final concentration of eccrine sweat that reaches the skin surface. Eccrine glands play an important role in the regulation of body temperature. They are distributed over the entire body skin, but are highest in density on the palms and soles. Eccrine glands secrete a salty solution, which reaches the skin pores via an eccrine duct. Eccrine sweat is a sterile electrolyte solution primarily containing sodium chloride, potassium and bicarbonate, with smaller quantities of various other components such as glucose and antimicrobial peptides. Sweat rate differs depending on site on the body, the degree of thermal or physical stress, and between individuals. Under maximal stimulation, the body can produce 3 liters of eccrine sweat per hour.

How well did you know this?
1
Not at all
2
3
4
5
Perfectly
6
Q

6- A mother presents to your pediatric dermatology clinic for further evaluation of her daughter, who has red gums, episodic hair loss, and rough perifollicular papules on the arms and thighs. She reports a family history of similar findings. After a thorough assessment of this patient, you conclude the patient has:

A. Pili annulati
B. Pili trianguli et canaliculi
C. Hereditary mucoepithelial dysplasia
D. Wooly hair
E. None of the above

A

Correct choice: C. Hereditary mucoepithelial dysplasia

Explanation: Defects of the hair shaft include hereditary mucoepithelial dysplasia which is characterized by red gums, episodic hair loss, and keratosis pilaris. Pili annulati is known as banded hair. Uncombable hair syndrome results in triangle-shaped hair. Naxos syndrome is characterized by wooly hair, hyperkeratosis of the palms & soles, and cardiac arrhythmias.

How well did you know this?
1
Not at all
2
3
4
5
Perfectly
7
Q

7- Occlusive moisturizers prevent evaporative water loss to the environment by placing an oily substance on the skin surface through which water cannot penetrate. This replenishes the stratum corneum by water movement from the viable epidermal and dermal layers. There are many different classes of chemicals that can function as occlusive moisturizers. What is the most effective occlusive moisturizer?

A. Glycerin
B. Petrolatum
C. Vegetable oil
D. Dimethicone
E. Ceramide

A

Correct choice: B. Petrolatum

Explanation: The most effective occlusive moisturizer is petrolatum, since it reduce transepidermal water loss by 99%. Petrolatum allows barrier repair while permeating throughout the interstices of the stratum corneum.
a- Glycerin: Humectants are substances that attract moisture; they include: glycerin, honey, sodium lactate, urea, propylene glycol, sorbitol, pyrrolidone carboxylic acid, gelatin, hyaluronic acid, and some vitamins and proteins. Some astringents designed for dry skin contain a humectant liquid moisturizer, such as propylene glycol or glycerin, and skin soothing agents, such as allantoin, guaiazulene or quaternium-19. Glycerin is not the most effective occlusive moisturizer.

c- Vegetable oil: There are many different classes of chemicals that can function as occlusive moisturizers, such as vegetable oils (castor oil, corn oil, grape seed oil, soybean oil). Vegetable oil and wax is a moisturizer type that composes castor oil, corn oil, ozokerite, beeswax, paraffin and carnauba wax. There are many different classes of chemicals that can function as occlusive moisturizers such as vegetable oil. Vegetable oil is not the most effective occlusive moisturizer.

d - Dimethicone: There are many different classes of chemicals that can function as occlusive moisturizers such as silicones (dimethicone, cyclomethicone). Dimethicone and ceramides are moisturizer types that compose water, petrolatum, dimethicone, and ceramics. Dimethicone is not the most effective occlusive moisturizer.
e - Ceramide: There are many different classes of chemicals that can function as occlusive moisturizers such as sterols (cholesterol and ceramides). Dimethicone and ceramides are moisturizer types that compose water, petrolatum,dimethicone, and ceramics. Ceramides are not the most effective occlusive moisturizers.

How well did you know this?
1
Not at all
2
3
4
5
Perfectly
8
Q

8- What structure is indicated by the arrow in this Mohs frozen section?

A. Erector pili muscle
B. Nerve
C. Hair shaft
D. Sebaceous gland
E. Vein

A

correct choice: A. Erector pili muscle

Explanation: A collection of elongated smooth muscle cells at a 45 degree angle in associated with a hair follicle is consistent erector pili muscle. Nerves are a bundle of spindled cells and are typically not seen in the upper dermis, and are not at a 45 degree angle, nor in connection with a hair follicle. Veins consistent of endothelial cells that would be lining a lumen; the lumen may appear collapsed. A hair shaft should be surrounded by various layers and sheaths, and is not made up of spindle cells. Sebaceous glands a a round collection of lobulated cells, not spindled cells, and are seen in association with a hair follicle.

How well did you know this?
1
Not at all
2
3
4
5
Perfectly
9
Q

9- The desmosomal connections of the epidermis are dependent on which of the following ions?

A. Iron
B. Zinc
C. Selenium
D. Calcium
E. Sodium

A

correct choice: D. Calcium

Explanation: The desmosomal connections in the epidermis are calcium dependent. The other options are not required for these connections. Desmogleins and desmocollins are cadherins (calcium-dependent adherence molecules)

How well did you know this?
1
Not at all
2
3
4
5
Perfectly
10
Q

10- A patient presents with a chronic pruritic papular eruption in both axillae. You suspect she may have Fox-Fordyce disease, due to an occlusion of apocrine ducts. Which of the following is also true of apocrine glands?

A. Parasympathetic innervation
B. Not present at birth
C. Stains with S100 and keratin
D. Denervation abolishes response to emotive stimuli
E. Distributed over the entire cutaneous surface

A

Correct choice: C. Stains with S100 and keratin

Explanation: Apocrine glands are innervated by sympathetic fibers. They are present at birth but do not enlarge until hormonal stimulation from puberty. Apocrine glands stain positive for S100, keratin, CEA, and lysozyme. Secretion is in response to emotive stimuli through action of epinephrine/norepinephrine. Denervation does not abolish this response although apocrine sweating requires intact nerve supply. Apocrine sweat glands are confined to certain anatomic locations (axillae, anogenital region, periumbilical region, areolae, nipples, vermilion border of the lip).

How well did you know this?
1
Not at all
2
3
4
5
Perfectly
11
Q

11- As the shown wound heals and matures, which cell type is most important for wound contraction?

A. Macrophages
B. Neutrophils
C. Fibroblasts
D. Myofibroblasts E.Endothelialcells

A

correct choice: D. Myofibroblasts

Explanation: Myofibroblasts are modified fibroblast with smooth muscle like features. Myofibroblasts play the important role in the contraction of the wound that occurs during the proliferation phase. The contraction is considered one of the important events in wound healing because it results in the closure of the wound. The correct balance between too little contraction, which leads to non-healing wounds, and too much contraction, which leads to contractures, is important for optimal healing. The other cell types do not play a large role in wound contraction.

How well did you know this?
1
Not at all
2
3
4
5
Perfectly
12
Q

12- Which type of collagen is the first to be deposited in a healing wound?

A. Type I
B. Type II
C. Type III
D. Type IV
E. Type V

A

Correct choice: C. Type III

Explanation: During the proliferative phase of wound healing, granulation tissue is formed, which relies on presence of fibronectin. Fibronectin is then replaced by collagen type III, and ultimately by

collagen type I. Wound contraction then ensues during the second week of wound healing, a process that is mediated by myofibroblasts. The remaining answer choices are incorrect.

How well did you know this?
1
Not at all
2
3
4
5
Perfectly
13
Q

13- Which of the options below, per high concentrations noted in recent studies, is the most important factor in the pathogenesis of SJS/TEN?

A. Granzyme B
B. TNF-β
C. IL-8
D. Caspase
E. IL-3

A

Correct choice: A. Granzyme B

Explanation: While the pathogenesis of SJS and TEN is not exactly understood, Granzyme B has been shown to be a key factor due to high concentrations present in affected skin. TNF-alpha, perforin and FasL have also been shown to be key to the development of the adverse drug dermatosis. The other cytokines listed are not as critical in the development of SJS/TEN.

How well did you know this?
1
Not at all
2
3
4
5
Perfectly
14
Q

14- This patient presents with chronic swelling of the lower extremities, and circumferential lesions on both lower legs as shown in this image. What is the most likely diagnosis?

A. Chronic venous stasis
B. Elephantiasis nostras verrucosa
C. Deep fungal infection
D. Peripheral arterial disease
E. Pigmented purpura dermatosis (Schamberg’s disease)

A

Correct choice: B. Elephantiasis nostras verrucosa

Explanation: Elephantiasis nostras verrucosa is complication from chronic ymphedema. It commonly affects distal feet and lower extremities leading to verrucous changes with papillomatosis and hyperkeratosis. The other options would not lead to circumferential verrucous lesions on both lower extremities.

How well did you know this?
1
Not at all
2
3
4
5
Perfectly
15
Q

15- A child with a stahylococcal infection has an exfoliative dermatitis. The pathogenesis of this eruption is similar to what disease?

A. Paraneoplastic pemphigus
B. Epidermolysis bulls simplex
C. Scarlet fever
D. Pemphigus foliaceous
E. Toxic epidermal necrolysis

A

Correct choice: D. Pemphigus foliaceous

Explanation: Staphylococcal scalded skin syndrome (SSSS) is descibed, a staphylococcal infection that has exfoliative properties, and the pathogenesis involves serine proteases binding desmoglein 1. This explains the clinical similarities between pemphigus foliaceous and SSSS. The other answer choices are not similar in pathogenesis to that of staphylococcal scalded skin syndrome.

How well did you know this?
1
Not at all
2
3
4
5
Perfectly
16
Q

16- Botulinum toxin acts by reducing the amount of acetylcholine release from nerve endings and can be targeted for use on sweat glands for hyperhidrosis. Which of the following sites has the greatest density of these sweat glands?

A. Nipple
B. Clitoris
C. Labia minora
D. External auditory canal
E. Vermilion lips

A

correct choice: A. Nipple

Explanation: A is correct. The question refers to eccrine glands, which are innervated by postganglionic sympathetic fibers and use acetylcholine as their neurotransmitter. They are targeted by botulinum toxin for hyperhidrosis (in contrast to apocrine glands; innervation of these glands is less clear). All of the following sites except for the nipple (A) lack eccrine glands.

How well did you know this?
1
Not at all
2
3
4
5
Perfectly
17
Q

17- An 80-year-old man with Parkinson’s disease developed a new pruritic eruption with these cutaneous findings. What is the antigenic target of the autoantibodies generated in this condition?

A. NC16A domain of BPAg1
B. NC16A domain of BPAg2
C. C-terminal of BPAg1
D. C-terminal of BPAg2
E. alpha-6-beta-4 integrin

A

Correct choice: B. NC16A domain of BPAg2

Explanation: Bullous pemphigoid is the most common immunobullous disorder and presents with tense vesicles and bullae and pruritus. There has been as association of neurologic disorders, such as Parkinson’s disease and dementia, with bullous pemphigoid. Autoantibodies are most commonly directed against the NC16A domain of BPAg2, also known as BP180 or collagen XVII.

How well did you know this?
1
Not at all
2
3
4
5
Perfectly
18
Q

18- Which of the following absorbs ultraviolet radiation?
A. Filaggrin
B. Loricrin
C. Involucrin
D. Urocanic acid
E. Laminin 332

A

Correct choice: D. Urocanic acid

Explanation: Urocanic acid is a chromophore that absorbs ultraviolet radiation (UVR) in the stratum corneum.This question assesses the examinee’s basic science knowledge of the function of skin proteins and their breakdown products. In the stratum corneum, urocanic acid (choice 4) functions as a chromophore that absorbs UVR and also mediates UV-induced immunosuppression. None of the remaining answer choices absorbs UVR. Urocanic acid is a degradation product of filaggrin (choice 1). Filaggrin cements the keratin filaments composing the stratum corneum; it is mutated in atopic dermatitis and ichthyosis vulgaris. Loricrin (choice 2) is the most plentiful component of the cornified envelope. Involucrin (choice 3), another component of the cornified envelope, is cross- linked to loricrin by transglutaminase 1. Laminin 332 (choice 5), found within the lamina lucida and densa, connects anchoring filaments to collagen VII.

How well did you know this?
1
Not at all
2
3
4
5
Perfectly
19
Q

19- Which of the following is NOT a degranulating stimulus for the largest cells seen in the image?

A. Ibuprofen
B. Fentanyl
C. Captopril
D. Vancomycin
E. Polymyxin B

A

Correct choice: C. Captopril

Explanation: The largest cells in the image are mast cells, which can be identified based on their size and presence of many intracellular granules. Captopril, and ACE-inhibitor, is not a degranulating stimulus for mast cells. The remaining answer choices are all mast cell degranulating stimuli, thus patients with mastocytosis should be counseled to avoid these medications if possible.

How well did you know this?
1
Not at all
2
3
4
5
Perfectly
20
Q

20- Which of the following medications is not concentrated in the eccrine glands?

A. Cyclophosphamide
B. Doxycycline
C. Cephalexin
D. Ciprofloxacin
E. Cytarabine

A

Correct choice: B. Doxycycline

Explanation: The above listed drugs, as well as beta-lactam antibiotics, antifungals such as ketoconazole and griseofulvin, are known to be secreted into eccrine sweat ducts. This feature may explain the development of neutrophilic eccrine hidradenitis and eccrine squamous syringometaplasia in the context of chemotherapy. Doxycycline does not accumulate in eccrine glands.

How well did you know this?
1
Not at all
2
3
4
5
Perfectly
21
Q

21- Which of the following is a conservative and effective treatment for chondrodermatitis nodularis helicis?

A. Mohs micrographic surgery
B. Aggressive cryodestruction with liquid nitrogen
C. TCA peel
D. Cushion pillow support on either side of the lesion to reduce focal pressure
E. Cantharidin application

A

Correct choice: D. Cushion pillow support on either side of the lesion to reduce focal pressure

Explanation: Chondrodermatitis nodularis helices presents as a tender nodule usually on the helix or antihelix of the ear. It is thought to be a result of pressure to the local, affected area. Multiple treatment modalities have been noted with conservative, effective treatment seen with cushion pillow application to reduce the focal pressure. The other options listed are either too invasive (Mohs, aggressive liquid nitrogen) or are less effective (TCA peels and cantharidin).

How well did you know this?
1
Not at all
2
3
4
5
Perfectly
22
Q

22- At any one time, the approximate proportion of hair follicles in anagen phase is:

A. 9%
B. 90%
C. 1%
D. 60%
E. 95%

A

Correct choice: B. 90%

Explanation: Most (~90%) hair follicles are in anagen (growth) phase. The longer a hair follicle is in anagen phase, the longer the hair can grow. Hairs of the scalp grow approximately 0.4 mm per day, and thus the date of one’s next hair cut can be accurately calculated. Approximately 1% of hairs are in catagen (transitional) phase, whereas ~9% of hairs are in telogen (resting) phase. The remaining answer choices (60% and 95%) are distractors.

How well did you know this?
1
Not at all
2
3
4
5
Perfectly
23
Q

23- All of the following are increased in psoriasis EXCEPT:

A. Ornithine decarboxylase
B. Keratin 6
C. Loricrin
D. Involucrin
E. Keratin 16

A

Correct choice: C. Loricrin

Explanation: Loricrin is the major protein component of the cell envelope. It is decreased in psoriasis. The remaining answer choices are increased in psoriasis.

How well did you know this?
1
Not at all
2
3
4
5
Perfectly
24
Q

24- Detection of immunoglobulins, complement, or both at the basement membrane zone in lesional skin by direct immunofluorescence occurs in approximately half of cases of which dermatologic condition?

A. Neonatal lupus erythematosus
B. SLE
C. Erythema multiforme

D. Mixed connective tissue disease
E. Lichen planus

A

Correct choice: A. Neonatal lupus erythematosus

Explanation: The key diagnostic sign of neonatal lupus erythematosus (NLE) is the erythematous and annular clinical appearance of lesions. The histopathologic findings may be subtle in NLE compared with discoid or subacute cutaneous lupus erythematosus. Direct immunofluorescence testing may be useful in supplementing the histopathologic evaluation. Detection of immunoglobulins, complement, or both at the basement membrane zone in lesional skin by immunofluorescence occurs in approximately half of cases; a negative study does not preclude the diagnosis of NLE syndrome. Clinical and serologic evaluation of both the infant and the mother are important in establishing a diagnosis of NLE syndrome, particularly when histopathologic findings are subtle.
DIF (direct immunofluorescence) of lichen planus is positive in the vast majority, with granular DEJ (dermal-epidermal junction) deposition and IgM and fibrinogen staining within cytoid bodies in the superficial dermis. In MCTD DIF shows IgG deposits within epidermal cell nuclei, and rarely along the DEJ. In SLE (systemic lupus erythematosus), while serology is more reliable, DIF can show DEJ deposition in the lupus band test. DIF of erythema multiforme shows immunoglobulin within superficial vessel walls, DEJ, and cytoid bodies.

How well did you know this?
1
Not at all
2
3
4
5
Perfectly
25
Q

25- A 16-year-old female presents with severe scarring, cystic acne. Which of the following immunologic factors are involved in the pathogenesis of acne?

A. TRPV channels
B. TLR-2
C. MMPs
D. IL-2
E. KLK5

A

Correct choice: B. TLR-2

Explanation: TLR-2 is a key aspect in the pathogenesis of acne. The other options listed are not key parts of the pathogenesis of acne, some are specifically involved in the pathogenesis of rosacea (KLK5, MMPs and TRPV channels).

How well did you know this?
1
Not at all
2
3
4
5
Perfectly
26
Q

26- All of the following locations have apocrine glands except:

A. Internal nares
B. External ear canal
C. Areola
D. Anogenital region
E. Periumbilical region

A

Correct choice: A. Internal nares

Explanation: Apocrine glands are found in the axillae, anogenital region, external ear canal, eyelids, breast, periumbilical region. Apocrine glands produce an oily fluid rich in triglycerides and fatty acids – subsequent colonization by anaerobic bacteria results in body odor.
Apocrine sweat glands are androgen-dependent for their development and have an unclear function in humans; primary locations are the axillae, anogenital region, periumbilical region and nipples Apocrine glands, whose apical portion (acrosyringium) drains into terminal hair follicles, continuously secrete a sterile odorless viscous fluid that is rich in precursors of odoriferous substances.

How well did you know this?
1
Not at all
2
3
4
5
Perfectly
27
Q

27- Subcorneal pustular dermatosis type IgA pemphigus is caused by a defect in what antigen?

A. Desmoglein 3
B. Desmoglein 1
C. BPAg1
D. Desmocollin 1
E. BPAg2

A

Correct choice: D. Desmocollin 1
Explanation: IgA pemphigus has two types subcorneal pustular dermatosis type which is due to ab to desmocollin 1 and intraepidermal n eutrophilic IgA dermatosis which is due to ab to desmoglein 3 and 1. Sneddon- Wilkinson disease, also known as sub-corneal pustular dermatosis has an unknown etiology. Desmoglein 1 defect is seen in pemphigus foliaceous and staph scalded skin syndrome. Desmoglein 3 defect is seen in pemphigus vulgaris and intraepidermal neutrophilic IgA dermatosis. BPAg1 is seen in paraneoplastic pemphigus and bullous pemphigoid. BPAg2 is seen in bullous pemphigoid, cicatricial mucous membrane pemphigoid and linear IgA disease.

IgA pemphigus (or immunoglobulin A pemphigus) is an autoimmune blistering disorder. It is also called intercellular IgA dermatosis among other names.
IgA pemphigus has two major subtypes:
1-Subcorneal pustular dermatosis (SPD) type 2- Intraepidermal neutrophilic (IEN) type.
The clinical features of IgA pemphigus include blisters, pustules, erythema, erosions and vegetating lesions. The first signs are flaccid vesicles and pustules. The vesicles and pustules rupture to form erosions and crusted plaques. The eruption tends to follow an annular pattern.
IgA pemphigus appears to favor the trunk, upper and lower extremities, axillae, and groin. Mucosal involvement is infrequent. The exact cause of IgA pemphigus is unclear. IgA autoantibodies bind to desmogleins or desmocollins, the cells responsible for adhesion between cells. In SPD-type IgA pemphigus, the autoantigen is described as desmocollin 1, one of the desmosomal cadherins, the glycoproteins that maintain the shape of a cell. In at least some cases of IEN-type IgA pemphigus, the autoantigens are desmoglein 1, desmoglein 3, and an unspecified transmembrane protein.

How well did you know this?
1
Not at all
2
3
4
5
Perfectly
28
Q

28- What is the most abundant amino acid in collagen?

A. Leucine
B. Proline
C. Glycine
D. Hydroxyproline
E. Hydroxylysine

A

Correct choice: C. Glycine

Explanation: Collagen is composed of 3 chains of amino acids combined into a triple helix configuration. It contains Gly-x-y repeats (glycine is always the 3rd residue). Thus, glycine is the most abundant amino acid in collagen. The remaining answer choices are not the most abundant amino acids in collagen.

How well did you know this?
1
Not at all
2
3
4
5
Perfectly
29
Q
  1. Which of the following medications is concentrated in the eccrine glands?

A. Cyclophosphamide
B. Cytarabine

C. Ciprofloxacin
D. Cephalexin
E. All of the answers are correct

A

Correct choice: E. All of the answers are correct

The above listed drugs, as well as beta-lactam antibiotics, antifungals such as ketoconazole and griseofulvin, are known to be secreted into eccrine sweat ducts. This feature may explain the development of neutrophilic eccrine hidradenitis and eccrine squamous syringometaplasia in the context of chemotherapy.

How well did you know this?
1
Not at all
2
3
4
5
Perfectly
30
Q
  1. Which element is necessary for function of matrix metalloproteinases?

A. Iron
B. Nitrogen
C. Manganese
D. Magnesium
E. Zinc

A

Correct choice: E. Zinc

Matrix metalloproteinases are required for normal tissue architecture and normal turnover of the extracellular matrix. All of them have zinc at the active site and require octahedral binding of calcium ions to maintain structural integrity.

How well did you know this?
1
Not at all
2
3
4
5
Perfectly
31
Q
  1. What is the most important cell for wound healing?

A. Fibroblasts
B. Neutrophils
C. Macrophages
D. Lymphocytes
E. Mast cells

A

Correct choice: C. Macrophages

Macrophages are the most important cell for wound healing. They secrete TGFs, cytokines, IL-1, TNF, and PDGF. Neutropenic or lymphopenic patients do not have impaired wound healing, whereas macrophage-deficient (quantity or function) patients heal poorly. Neutrophils however are the first cell type to flood the wound during phase I of inflammation (within first 6-8 hours); this movement is facilitated by TGF-beta. Fibroblasts migrate into the wound by stage II (granulation) where they produced glycosaminoglycans and fibronectin.

How well did you know this?
1
Not at all
2
3
4
5
Perfectly
32
Q
  1. Which of the following is true in or associated with Apert Syndrome?

A. Secondary to a mutation in FGFR3
B. AR
C. Acne localized to buttocks and thighs
D. Cutaneous/ocular depigmentation
E. Synostoses

A

Correct choice: E. Synostoses

Apert Syndrome is an AD syndrome secondary to a mutation in FGFR2. Also known as acrocephlosyndactyly, this condition includes synostoses of the hands, feet, back and skull as well as generalized acne. Mosaicism of this gene causes nevus comedonicus. There is associated cutaneous and ocular hypopigmentation.

How well did you know this?
1
Not at all
2
3
4
5
Perfectly
33
Q
  1. What is the most abundant collagen found on fetal skin?

A. Type I Collagen
B. Type II Collagen
C. Type III Collagen
D. Type IV Collagen
E. Type VII Collagen

A

Correct choice: C. Type III Collagen

Type III collagen is found in the fetal skin. It is also present in the gastrointestinal tract, blood vessels, and the basement membrane. A defect in this collagen results in the Ehlers-Danlos, vascular type.

How well did you know this?
1
Not at all
2
3
4
5
Perfectly
34
Q
  1. Which component of hair is positive for citrulline?

A. Outer root sheath
B. Inner root sheath
C. Cortex
D. Glassy vitreous layer
E. Medulla

A

Correct choice: B. Inner root sheath

The inner root sheath stains red because it contains citrulline. The cortex and medulla along with the cuticle make up the hair shaft. The outer root sheath is the most peripheral cellular structure. The glassy vitreous layer is the basement zone equivalent of hair and is the outermost layer.

How well did you know this?
1
Not at all
2
3
4
5
Perfectly
35
Q
  1. The major protein component of the cornified envelope is:
    A. Envoplakin
    B. Desmoplakin
    C. Plectin
    D. Loricrin
    E. Transglutaminase
A

Correct choice: D. Loricrin

Loricrin is the major component of the cornified envelope (CE). The proteins of the CE are synthesized in the spinous and granular layers. The CE is primarily a protein/lipid polymer formed within the differentiating layer of keratinocytes. The CE eventually exists outside of the cornified cells after the granular cell undergoes a programmed destruction (apoptosis). Self-destructing granular cells are called transition cells.

How well did you know this?
1
Not at all
2
3
4
5
Perfectly
36
Q
  1. The most common type of pityriasis rubra pilaris in childhood is type:

A. I
B. II
C. III
D. IV
E. V

A

Correct choice: D. IV

Type IV, or circumscribed juvenile, accounts for 25% of total PRP cases. The most common type is Type I, classical adult, which accounts for 55% of cases. Types II, III, and V account for less than 10% each.

How well did you know this?
1
Not at all
2
3
4
5
Perfectly
37
Q
  1. Elastic fibers contain the specific amino acids:
    A. Lysine and proline
    B. Leucine and isoleucine
    C. Alanine and phenylalanine
    D. Desmosine and isodesmosine
    E. Glycine and proline
A

Correct choice: D. Desmosine and isodesmosine

Desmosine and isodesmosine are the typical amino acids of elastic fibers. Elastic fibers are comprised of elastin that is wrapped by fibrillin microfibrils. Elastic fibers form a complex meshwork extending from the lamina densa of the dermo-epidermo junction through the dermis. Elastic fibers return the skin to a normal shape after being stretched.

How well did you know this?
1
Not at all
2
3
4
5
Perfectly
38
Q
  1. Anchoring fibrils are primarily composed of:

A. Type I collagen
B. Type III collagen
C. Type IV collagen
D. Type VII collagen
E. Type II collagen

A

correct choice: D. Type VII collagen.

Anchoring fibrils are found in the sublamina densa and are made up of collagen type VII. This collagen type is mutated in dystrophic epidermolysis bullosa, and targeted in epidermolysis bullosa acquisita and bullous lupus erythematosus.

How well did you know this?
1
Not at all
2
3
4
5
Perfectly
39
Q
  1. A child presents with a 1 cm yellow-red nodule on the face. Pathology shows Touton giant cells. What is the most frequent site of extracutaneous involvement in this disease?

A. Eye
B. Lung
C. Bone
D. CNS
E. Visceral

A

Correct choice: A. Eye

The eye is the most frequent site of extracutaneous juvenile xamthogranuloma. The second most common site of extracutaneous disease is the lungs. Ocular involvement is typically unilateral.

How well did you know this?
1
Not at all
2
3
4
5
Perfectly
40
Q
  1. Keratinocytes are derived from which of the following:

A. Endoderm
B. Mesoderm
C. Ectoderm
D. Neural Crest
E. Bone marrow precursors

A

Correct choice: C. Ectoderm .

As implied by the root ‘ecto’, a prefix meaning “outer”, the keratinocytes of the epidermis are derived from the ectoderm. The other layers do contribute cell populations that are present in the skin.

How well did you know this?
1
Not at all
2
3
4
5
Perfectly
41
Q
  1. Which epidermal layer do pilar cysts generally not have a:

A. Stratum corneum
B. Stratum granulosum
C. Stratum spinosum
D. Stratum basale
E. Stratum lucidum

A

Correct choice: B. Stratum granulosum

Pilar cysts do not have a granular layer, they do have all the other layers.

How well did you know this?
1
Not at all
2
3
4
5
Perfectly
42
Q
  1. Fragmentation and/or loss of elastic fibers in not seen in:

A. Cutis laxa
B. Marfan’s syndrome
C. Anetoderma
D. Psuedoxanthoma elasticum
E. Buschke-Ollendorf syndrome

A

Correct choice: E. Buschke-Ollendorf syndrome

Cutis laxa results from decreased desmosine and lysyl oxidase and demonstrates fragmentation and loss of elastic fibers. Marfan’s sydnrome results from decreased fibrillin I and demonstrates fragmentation of elastic fibers. Pseudoxanthoma elasticum demonstrates increased glycosaminoglycans on elastic fibers and the accumulation of fragmented and calcified elastic fibers. Anetoderma has decreased desmosine and demonstrates loss and fragmentation of elastic fibers. Buschke- Ollendorf syndrome exhibits increased desmosine and an increased amount of thickened elastic fibers.

How well did you know this?
1
Not at all
2
3
4
5
Perfectly
43
Q
  1. Which of the following protein plays a major role in wound healing?

A. Uncein
B. Fibronectin

C. Nidogen
D. Entactin
E. Band-6 protein

A

Correct choice: B. Fibronectin

Fibronectin is a key player in wound healing, initially secreted by myofibroblasts. The bed/matrix of fibronectin provides an adherent base for migration into the wound, provides scaffolding for collagen fibrils and mediates wound contraction. Band-6 protein is a constituent of the desmosomes intracellular plaque. Entactin and nidogen are synonyms and are found in the dermal-epidermal junction, binding to laminin 1s alpha chain. Uncein is associated with anchoring filaments.

How well did you know this?
1
Not at all
2
3
4
5
Perfectly
44
Q
  1. Keratinocytes have been shown to secrete all of the following cytokines except:

A. IL-1
B. IL-6
C. IL-8
D. TNF-alpha
E. IL-2

A

Correct choice: E. IL-2

Keratinocytes have been shown to secrete all of the above cytokines, except IL-2, IL- 4, and IFN- gamma.

How well did you know this?
1
Not at all
2
3
4
5
Perfectly
45
Q
  1. The microflora of pilosebaceous unit consist of which of the following:

A. Pityrosporum ovale
B. Staphylococcus aureus
C. Escherichia coli
D. Pseudomonas aeruginosa
E. Corynebacterium diphtheriae

A

Correct choice: A. Pityrosporum ovale

All the above bacteria and fungi are found within sebaceous glands; the Malasssezia spp. and P. ovale are found within the acroinfundibulum, S. epidermidis is found within the midinfundibulum, and Propionibacterium spp. deep within the follicle.

How well did you know this?
1
Not at all
2
3
4
5
Perfectly
46
Q
  1. Regarding dermal-epidermal junction, which of the following statements is true:

A. There are no anchoring filaments in lamina lucida
B. Lamina fibroreticularis lies above lamina densa
C. Lamina fibroreticularis comprises of anchoring fibrils and the elastic microfibrils
D. Blood vessels cross the dermal-epidermal junction to reach the epidermis
E. Lamina lucida is an electron-dense layer

A

Correct choice: C. Lamina fibroreticularis comprises of anchoring fibrils and the elastic microfibrils

Lamina fibroreticularis comprises of anchoring fibrils and the elastic microfibrils. Lamina fibroreticularis lies below lamina densa. Blood vessels don’t cross the dermal-epidermal junction to reach the epidermis. Basement membrane contain both: 1)Lamina basale which include a.lamina lucida and b.lamina densa ( which contain collagene IV & other glycoproteins like laminins,fibronectin etc.) 2)Lamina fibroreticularis contain collagen type III and anchoring fibrils (collagen type VII).

How well did you know this?
1
Not at all
2
3
4
5
Perfectly
47
Q
  1. Direct immunofluorescence staining of intercellular spaces and the basement membrane zone, in combination, is seen in:

A. Paraneoplastic pemphigus
B. Anti-epiligrin pemphigoid
C. Pemphigus vegetans
D. Pemphigus foliaceous
E. IgA pemphigus

A

Correct choice: A. Paraneoplastic pemphigus

Paraneoplastic pemphigus and drug-induced pemphigus demonstrate direct immunofluorescence staining of the intercellular space and the BMZ, in combination. Anti-epiligrin pemphigoid demonstrates BMZ staining (dermal staining on salt-split skin). Pemphigus vegetans, pemphigus foliaceous, and IgA pemphigus all show intercellular space deposition without staining of the BMZ.

How well did you know this?
1
Not at all
2
3
4
5
Perfectly
48
Q
  1. Keratohyalin granules contain:

A. Desmoplakin
B. Envoplakin and Keratin 6
C. Profilaggrin and loricrin
D. Numerous Golgi apparati
E. Involucrin

A

Correct choice: C. Profilaggrin and loricrin

Keratohyalin granules are found in the stratum granulosum (the granular layer), and contain the proteins profilaggrin and loricrin. Profilaggrin is
converted to filaggrin during the transformation of the granular layer to the cornified layer. This is a calcium-dependent process. Loricrin comprises 75% of the cornified envelope�s mass.

How well did you know this?
1
Not at all
2
3
4
5
Perfectly
49
Q
  1. Which eponym describes vestigial lines of pigmentary demarcation?

A. Futcher lines
B. Wallace’s lines
C. Langer’s lines
D. Lines of Blaschko
E. Dermatome

A

Correct choice: A. Futcher lines

Futcher lines are vestigial lines in which the dorsal surface has more melanocytes than ventral surface. Wallace’s lines are the well-demarcated lines around the margin of the foot and hand.

How well did you know this?
1
Not at all
2
3
4
5
Perfectly
50
Q
  1. Sebaceous glands are located in each of the following locations except:

A. Nipple
B. Labia minora
C. Palms
D. Eyelids
E. Buccal mucosa

A

Correct choice: C. Palms

Sebaceous glands secrete their contents in a holocrine fashion and are primarily under the influence of androgens. They secrete triglycerides, phospholipids, esterified cholesterol, waxes but not free cholesterol. They may be found in association with hair follicles or in some areas of modified skin such as the nipple/areola, labia minora, prepuce, vermilion border, and eyelids.

How well did you know this?
1
Not at all
2
3
4
5
Perfectly
51
Q
  1. All of the following pertain to Odland bodies EXCEPT:

A. Contain squalene
B. Are found intracellularly in upper level keratinocytes
C. Discharge their contents into the extracellular space at the junction of the granular and cornified layers
D. Establish a barrier to water loss
E. Mediate stratum corneum adhesion in conjunction with filaggrin.

A

Correct choice: A. Contain squalene

Odland bodies mediate stratum corneum adhesion in conjunction with FILAGGRIN. They discharge their contents into the extracellular space at the junction of the granular and horny layers establish a barrier to water loss. They are first found intracellularly in upper level keratinocytes and contain ceramides not squalene.

How well did you know this?
1
Not at all
2
3
4
5
Perfectly
52
Q
  1. Red or blonde hair pigmentation primarily results from:

A. The presence of eumelanin

B. The absence of melanin
C. The presence of pheomelanin
D. The reduced activity of tyrosinase
E. The reduced activity of DOPA dehydroxylase

A

Correct choice:C. The presence of pheomelanin

Hair color is determined by melanocytes. The melanocytic activity of follicular melanocytes is coupled to anagen � hair is only pigmented when it is growing. Pigment is produced in the matrix area of follicle, above the follicular papilla. Eumelanin is the pigment of brown/black hairs, and pheomelanin is the pigment of red/blonde hairs. Intensity of color is proportional to the amount of pigment. The absence of pigment produces white hair, and markedly reduced pigment produces gray hair.

How well did you know this?
1
Not at all
2
3
4
5
Perfectly
53
Q
  1. Epidermolysis bullosa simplex (EBS), Weber Cockayne type, is caused by what defect?

A. Collagen VII
B. Alpha-6-beta-4 integrin
C. Keratins 1 & 10
D. Keratins 5
E. Plectin

A

Correct choice: D. Keratins 5

All subtypes of EBS are caused by a defect in keratins 5 & 14 except EBS with muscular dystrophy which is caused by a defect in plectin. An alpha-6-beta-4 integrin defect is seen in junctional epidermolysis bullosa (JEB) with pyloric atresia. CollagenVII defect is seen in EB aquisita (EBA), Bart’s syndrome and dominant EB (DEB). Keratin 1 & 10 defects are not seen in any of the EB subtypes.

How well did you know this?
1
Not at all
2
3
4
5
Perfectly
54
Q
  1. Which of the following statements about elastic fibers is true?

A. Elastic fibers form 35% of the dry weight of the skin
B. Elastic fibers are 90% elastin wrapped in fibrillin

C. Collagen 1 is mutated in Marfan syndrome
D. Oxytalan fibers run parallel within the superficial papillary dermis
E. Elaunin fibers run perpendicular in thin bands within the reticular dermis

A

Correct choice: B. Elastic fibers are 90% elastin wrapped in fibrillin

Elastic fibers are responsible for much of the elasticity of the dermis. They are essentially 90% elastin wrapped in fibrillin. They form 4% of the dry weight of the skin. Fibrillin 1 is mutated in Marfan syndrome. Oxytalan fibers run PERPENDICULAR from the DEJ within the superficial papillary dermis. Elaunin fibers run parallell in thin bands within the reticular dermis.

How well did you know this?
1
Not at all
2
3
4
5
Perfectly
55
Q
  1. Eccrine glands are found in all the following areas of the body except:

A. Axillae
B. Palms
C. Labia minora
D. Scalp
E. Cutaneous lip

A

Correct choice: C. Labia minora

Eccrine glands are sweat glands enervated by cholinergic sympathetic nerves mediated by acetylcholine. They absent on modified skin which lacks appendages like the vermillion border, nail beds, glans penis, inner aspect of the prepuce and the labia minora.

How well did you know this?
1
Not at all
2
3
4
5
Perfectly
56
Q
  1. When evaluating a foreign body, which substance would be PAS negative and have no bifringence on polarizing microscopy?

A. Silica
B. Talc
C. Zinc
D. Aluminium
E. Wood splinters

A

Correct choice: D. Aluminium

Starch, cactus spines, and wood splinters are PAS positive. Silica, talc, zinc, and wood splinters are positive for bifringence on microscopy.

How well did you know this?
1
Not at all
2
3
4
5
Perfectly
57
Q
  1. Which of the following statements about the direct immunofluorescence pattern in lichen planus is correct?

A. The DIF is negative in the vast majority of cases
B. Deposition of IgG is within cytoid bodies in the superficial dermis
C. The DEJ deposition is granular
D. Deposition of fibrinogen is within cytoid bodies in the deep dermis
E. There is prominent deposition of IgM within the spinous layer of the epidermis.

A

Correct choice: C. The DEJ deposition is granular

The DEJ deposition is granular. The DIF is positive in the vast majority of cases. Deposition of IgM and fibrinogen is within cytoid bodies in the superficial dermis. There is no deposition of IgM within the spinous layer of the epidermis.

How well did you know this?
1
Not at all
2
3
4
5
Perfectly
58
Q
  1. Human sebum is distinguished from lipids of internal organs by the presence of:

A. Cholestrol
B. Cholestrol esters
C. Squalene
D. Wax esters
E. Glycerides

A

Correct choice: D. Wax esters

As human sebum exits the sebaceous gland, its major constituents are squalene, cholesterol, cholesterol esters, triglycerides, and wax esters. With passage through the hair follicle, triyglycerides in the sebum become hydrolyzed by bacterial enzymes, so that by the time the sebum reaches the skin surface, it contains free fatty acids, mono- and diglycerides in addition to the original components. Human sebum is distinguished by the presence of wax esters and squalene. The lipids of humaninternal organs contain no wax esters and little squalene. The squalene that is produced in internal organs is quickly converted to lanosterol and then to cholesterol, so it does not remain in its original form. Human sebaceous glands do not convert squalene to sterols.

How well did you know this?
1
Not at all
2
3
4
5
Perfectly
59
Q
  1. Melanocytes are derived from:

A. Bone marrow
B. Neural crest
C. Mesodermal precursors
D. Endodermal precursors
E. Yolk sac derived

A

Correct choice: B. Neural crest

Melanocytes are derived from neural crest precursors and migrate to the epidermis, hair matrix, retinal pigment epithelium, ear (stria vascularis), leptomeninges, and mucous membranes. The other options listed are incorrect and are not involved with melanocytes.

How well did you know this?
1
Not at all
2
3
4
5
Perfectly
60
Q
  1. Apocrine glands are found in all of the following areas of the body except:

A. Axillae
B. Breasts
C. Eyelid
D. Palms
E. Perineum

A

Correct choice: D. Palms

Apocrine glands operate by decapitation secretion and are activated by epinephrine and norepinephrine. They are located in a few distinct areas of the body, which include axillae, anogenital region, Moll’s glands of the eyelids, mammary glands of the breast and the ceruminous glands of the external auditory canal.

How well did you know this?
1
Not at all
2
3
4
5
Perfectly
61
Q
  1. Anagen effluvium is best described as:

A. An abrupt transition from anagen to catagen in rapidly dividing hair matrix cells
B. A cessation of mitotic activity in rapidly dividing hair matrix cells
C. An abrupt transition of telogen to anagen in resting hair matrix cells
D. A cessation of mitotic activity in resting hair matrix cells
E. A scarring alopecia affecting only anagen stage follicles

A

Correct choice: B. A cessation of mitotic activity in rapidly dividing hair matrix cells

Anagen effluvium results from an outside stimulus – most often an antimetabolite, chemotherapeutic drug – inducing an abrupt cessation of hair matrix cell mitotic activity. This process occurs within days to weeks of the stimulus, and is reversible with cessation of the drug therapy.

How well did you know this?
1
Not at all
2
3
4
5
Perfectly
62
Q
  1. Which of the following statements is true about eccrine glands?

A. Postganglionic sympathetic fibers with acetylcholine as the principal neurotransmitter
B. Postganglionic sympathetic fibers with norepinephrine as the principal neurotransmitter
C. Postganglionic parasympathetic fibers with acetylcholine as the principal neurotransmitter
D. Postganglionic parasympathetic fibers with norepinephrine as the principal neurotransmitter
E. Postganglionic sympathetic fibers with both norepinephrine and acetylcholine as the principal neurotransmitters

A

Correct choice: A. Postganglionic sympathetic fibers with acetylcholine as the principal neurotransmitter.

Eccrine glands are innervated by postganglionic sympathetic fibers with acetylcholine as the principal neurotransmitter. This explains why medications associated with anticholinergic side effects may be associated with hypohidrosis.

How well did you know this?
1
Not at all
2
3
4
5
Perfectly
63
Q
  1. The strength of a scar:

A. Is 5% at 1 week
B. Is 20% at 3 weeks
C. Is 70% at 1 year
D. All of these options are correct
E. None of these options are correct

A

Correct choice: D. All of these options are correct

A scar has 5% strength at 1 week, 20% at 3 weeks and 70% at 1 year. It will never recover strength to the level of pre-injury.

How well did you know this?
1
Not at all
2
3
4
5
Perfectly
64
Q
  1. The target for GABEB and bullous pemphigoid is:

A. Type XVII collagen
B. Type VII collagen
C. Laminin 5
D. Integrin subunit B4
E. Type XII collagen

A

Correct choice: A. Type XVII collagen

The target for GABEB and bullous pemphigoid is type XVII collagen or BPAG2. It is also the target protein in pemphigoid gestationis, CP and linear IgA.

How well did you know this?
1
Not at all
2
3
4
5
Perfectly
65
Q
  1. All mononuclear phagocytic cells in the dermis express:

A. CD3
B. CD6
C. CD34
d. CD68
E. CD20

A

Correct choice: B. CD6

CD6 and CD11c are expressed on all mononuclear phagocytic cells in the dermis. CD3 is a T-cell marker and CD20 is a B-cell marker. CD34 is expressed on mast cells and CD68 on macrophages.

How well did you know this?
1
Not at all
2
3
4
5
Perfectly
66
Q
  1. How soon does epithelialization begin after a skin wound occurs?

A. Minutes
B. Hours
C. 2 days
D. 4 days
E. 6 days

A

Correct choice: B. Hours

Re-epithelialization begins hours after an injury occurs. Keratinocytes from residual epithelial structures leapfrog each other. One to two days after injury, cells at the wound margin proliferate and begin to migrate into the
wound.

How well did you know this?
1
Not at all
2
3
4
5
Perfectly
67
Q
  1. Regarding sebaceous glands:

A. These glands are present at birth at their adult size
B. Size of the gland is proportional to the size of the associated hair follicle
C. Are always associated with a hair follicle
D. Are found everywhere on the skin except palms and soles
E. Are unilobular glands

A

Correct choice: D. Are found everywhere on the skin except palms and soles

Sebaceous glands are found everywhere on the skin except on the palms and soles. They are multilobular, emptying into the sebaceous duct. Most sebaceous glands are associated with hair follicles, but free glands exist, especially on the lip (Fordyce spots), on the nipple/ areola(Montgomery�s tubercles), Meibomian glands of the eyelids and Tyson�s glands on the

genitalia. The size of the sebaceous glands are not related to the size of the associated follicle. The sebaceous gland enlarges at puberty in response to increased androgens.

How well did you know this?
1
Not at all
2
3
4
5
Perfectly
68
Q
  1. What is the major function of urocanic acid?

A. Bacteriocidal acid produced by stratum corneum
B. Primarily a UVB filter
C. Primarily a UVA filter
D. Helps degrade free fatty acids
E. Aids in protecting the skin from dermatophytes

A

Correct choice: C. Primarily a UVA filter

Urocanic acid a by product of filaggrin degradation and has a peak absorbtion of 345 nm. It serves as a major UVA filter.

How well did you know this?
1
Not at all
2
3
4
5
Perfectly
69
Q
  1. The main collagen component of the basement membrane is:

A. Collagen IV
B. Collagen III
C. Collagen I
D. Tenascin-X
E. Collagen VII

A

Correct choice: A. Collagen IV

Collagen IV is the main collagen component of basement membranes. Collagen I is the main collagen of mature dermis, bone and tendon. Collagen III is found in fetal skin, blood vessels and intestines. Tenascin-X is mutated in some forms of Ehlers- Danlos syndrome and is not associated with the basement membrane. Collagen VII makes up anchoring fibrils and amnion.

How well did you know this?
1
Not at all
2
3
4
5
Perfectly
70
Q
  1. The main permeability barrier in the lamina densa is:

A. heparan sulfate proteoglycan
B. collagen IV
C. laminin 5
D. nidogen
E. alpha-6-beta-4 integrin

A

Correct choice: A. heparan sulfate proteoglycan

All of the listed proteins are present in the lamina densa except alpha-6-beta-4 integrin, which connects the hemidesmosome to laminin 5 in the lamina lucida. The heparin sulfate proteoglycans (perlecan) are negatively charged, thus serve as a permeability barrier. The other listed proteins do not serve this function in the lamina densa. Collagen IV is the main basic basement membrane scaffold. Defects in Collagen IV have been linked to Alport�s and Goodpasture�s syndrome.
Nidogen has a �dumbbell� shape and binds both laminins and collagen IV in the lamina densa.

How well did you know this?
1
Not at all
2
3
4
5
Perfectly
71
Q
  1. The desmosomal connections of the epidermis are dependent on which of the following ions?

A. Iron
B. Zinc
C. Selenium
D. Calcium
E. Sodium

A

Correct choice: D. Calcium

The desmosomal connections in the epidermis are calcium dependent. The other options are not required for these connections.

How well did you know this?
1
Not at all
2
3
4
5
Perfectly
72
Q
  1. Eulanin fibers:

A. Run parallel in bands within the superficial papillary dermis
B. Run perpendicular from the dermo-epidermal junction within the superficial papillary dermis
C. Run parallel in bands within the reticular dermis

D. Run perpendicular in bands within the deep dermis
E. Run perpendicular in bands within the reticular dermis

A

Correct choice: C. Run parallel in bands within the reticular dermis

Eulanin fibers are elastic fibers that have less elastin and more fibrillin and run parallel in thin bands within the reticular dermis. Oxytalin fibers contain no elastin and run perpendicular from the dermo-epidermal junction withinthe superficial papillary dermis. Elastic fibers turn over slowly in the skin, and are damaged by ultraviolet radiation.

How well did you know this?
1
Not at all
2
3
4
5
Perfectly
73
Q
  1. Meibomian glands are:

A. Eccrine glands localized to the vermillion border of the lips
B. Sebaceous glands found on the areola of the breast
C. Sebaceous glands found on the eyelids
D. Apocrine glands found in the anogenital regions
E. Apocrine glands found on the eylelids

A

Correct choice: C. Sebaceous glands found on the eyelids

Sebaceous glands enlarge at puberty in response to increased levels of androgens. They are holocrine glands. Meibomian glands are modified sebaceous glands foundin the eyelids. Free sebaceous glands not associated with hairs are found in the nipple and areola and are called Montgomery�s tubercles. Fordyce�s condition involves free sebaceous glands on the vermillion border of the lips and on the buccal mucosa. Sebaceous glands are found everywhere on the skin except the palms and
soles.

How well did you know this?
1
Not at all
2
3
4
5
Perfectly
74
Q
  1. Which of the following is true about melanosomes?

A. Spheroid melanosomes have concentric lamellae
B. Spheroid melanosomes synthesize brown-black eumelanin
C. Elliptical melanosomes have microvesicular structure
D. Elliptical melanosomes synthesize yellow or red pheomelanin.

E. The melanosomes are positioned after the Golgi apparatus in the secretory pathway.

A

Correct choice: E. The melanosomes are positioned after the Golgi apparatus in the secretory pathway.

Melanosomes are organelles related to lysosomes and are positioned after the Golgi apparatus in the secretory pathway. Elliptical melanosomes have concentric lamellae and synthesize brown-black eumelanin. Spheroid melanosomes have microvesicular structure and synthesize yellow or red pheomelanin.

How well did you know this?
1
Not at all
2
3
4
5
Perfectly
75
Q
  1. Which sebaceous gland is located on the eyelids in association with eyelashes?

A. Montgomery’s tubercles
B. Tyson’s glands
C. Zeis glands
D. Meibomian glands
E. Fordyce’s spots

A

Correct choice: C. Zeis glands

Zeis glands are sebaceous glands associated with eyelashes. Montgomery’s
tubercles are present on the areola, Tyson’s glands on the labia minora and glans, and Fordyce’s spots are located on the buccal mucosa.

How well did you know this?
1
Not at all
2
3
4
5
Perfectly
76
Q
  1. Which part of the nose is not innervated by V2 branch of Cranial nerve V?

A. nasal columella
B. nasal ala
C. nasal tip
D. nasal dorsum
E. all sensory of the nose is innervated by V2

A

Correct choice: C. nasal tip

This is innervated by the anterior ethmoidal branch of V1. The infraorbital nerve of V2 innervates the nasal ala. The nasopalantine branch of V2 innervates the columella.

How well did you know this?
1
Not at all
2
3
4
5
Perfectly
77
Q
  1. Tyrosinase is the enzyme that catalyzes the conversion of tyrosine to DOPA and DOPA to DOPAquinone. The enzyme contains which of the following ions?

A. Zinc
B. Copper
C. Selenium
D. Iron
E. Magnesium

A

Correct choice: B. Copper

Tyrosinase is a copper containing enzyme that is responsible for the conversion to tyrosine to DOPA and DOPA to DOPAquinone.

How well did you know this?
1
Not at all
2
3
4
5
Perfectly
78
Q
  1. In epidermolysis bullosa simplex, where on the blister does the signal localize on a salt split skin test?

A. Roof
B. Floor
C. Middle
D. Diffuse
E. No localization

A

Correct choice: A. Roof

In dystrophic EB the signal localizes to the roof of the blister. In junctional EB, the roof has BPAG2 while the floor has type IV collagen.

How well did you know this?
1
Not at all
2
3
4
5
Perfectly
79
Q
  1. The mechanism of action of ipilimumab can be described as

A. Inhibitor of CTLA-4
B. Antibody to CD27
C. Fusion protein binding B7
D. Antibody to CD 8 T cell
E. Inhibitor of CD27

A

Correct choice: A. Inhibitor of CTLA-4

Ipilimumab is an antibody to CTLA-4, this interaction causes inhibition of CTLA-4 which normally inhibits T cell co-activation by binding to B7 (which normally binds to CD28).

How well did you know this?
1
Not at all
2
3
4
5
Perfectly
80
Q
  1. Desmosine and isodesmosine are typical amino acids found in:

A. Collagen fibers
B. Anchoring fibril
C. Elastic fibers
D. Heparan sulfate
E. Anchoring plaques

A

Correct choice: C. Elastic fibers

Desmosine and isodesmosine are typical amino acids found in elastic fibers. They crosslink fibrillin. Anchoring fibrils are composed of collagen VII and collagen fibers and have the most typical amino acids of proline and hydroxyproline. Heparan sulfate do not typically contain these amino acids.

How well did you know this?
1
Not at all
2
3
4
5
Perfectly
81
Q
  1. A salt split skin DIF is performed on a biopsy taken adjacent to the skin lesions shown. Where would you expect staining to be seen?

A. Epidermal side
B. Dermal side
C. Epidermal and Dermal sides equally
D. In the lamina densa

E. In the anchoring

A

Correct choice: A. Epidermal side

The image shown is bullous pemphigoid. On salt split skin DIF exams, deposits are seen on the epidermal side of the split. If dermal deposits are seen, epidermolysis bullosa acquisita or anti- epiligrin pemphigoid are potential diagnoses.

How well did you know this?
1
Not at all
2
3
4
5
Perfectly
82
Q
  1. Homocystinuria is an autosomal recessive condition with findings including a marfanoid habitus, downward dislocation of the lens, cardiovascular disease and mental retardation. It is caused by a mutation in cystathionine beta-synthetase. What does this mutation in cystathionine beta-synthetase cause other than an accumulation of homocystine?

A. Abnormal crosslinking of collagen
B. Abnormal development of elastin fibers
C. Melanocyte death
D. Pigmentation of cartilage
E. Black urine

A

Correct choice: A. Abnormal crosslinking of collagen

Excess homocystine leads to abnormal crosslinking of collagen, with only ~1/3 of crosslinking activity compared to normal controls . The other listed findings are not features of homocystinuria, though hypopigmentation can be a feature.

How well did you know this?
1
Not at all
2
3
4
5
Perfectly
83
Q
  1. The following is a target in both junctional EB with pyloric atresia and ocular CP:

A. Integrin subunit B4
B. Laminin 5
C. Type VII Collagen
D. Type XVII collagen
E. BPAG2

A

Correct choice: A. Integrin subunit B4

Integrin subunit B4 is the target for both junctional EB with pyloric atresia and ocular CP.

How well did you know this?
1
Not at all
2
3
4
5
Perfectly
84
Q
  1. Direct immunofluorescence is of no value in the diagnosis of:

A. Neonatal LE
B. Lichen planus
C. Mixed connective tissue disease
D. SLE
E. Erythema multiforme

A

Correct choice: A. Neonatal LE

DIF is of no value in the diagnosis of scleroderma, morphea, and neonatal LE. DIF of lichen planus is positive in the vast majority, with granular DEJ deposition and IgM and fibrinogen staining within cytoid bodies in the superficial dermis. In MCTD DIF shows IgG deposits within epidermal cell nuclei, and rarely along the DEJ. In SLE, while serology is more reliable, DIF can show DEJ deposition in the lupus band test. DIF of erythema multiforme shows immunoglobulin within superficial vessel walls, DEJ, and cytoid bodies.

How well did you know this?
1
Not at all
2
3
4
5
Perfectly
85
Q
  1. Type 1 collagen is the most prevalent collagen in skin, accounting for 80% or more of the total collagen in the adult dermis. The next most predominant collagen in adult human dermis is:

A. Type II collagen
B. Type III collagen
C. Type IV collagen
D. Type VII collagen
E. Type XVII collagen

A

Correct choice: B. Type III collagen

Type 1 collagen is the most prevalent collagen in skin, accounting for 80% or more of the total collagen in the adult dermis. Type III collagen is the next most predominant collagen in human

dermis, accounting for approximately 10%. Type IV collagen is found in basement membranes. Type VII collagen is found in human dermis as anchoring fibrils. Type XVII collagen, also known as BPAG2, is an important transmembrane pro
tein in the basement membrane zone.

How well did you know this?
1
Not at all
2
3
4
5
Perfectly
86
Q
  1. The formation of granulation tissue depends on the presence of:

A. Neutrophils
b. Fibronectin
C. Collagen type I
D. Platelets
E. Collagen type IV

A

Correct choice: B. Fibronectin

Granulation tissue forms approximately four days after injury. It is composed of new capillaries, macrophages, fibroblasts, and blood vessels. The formation is dependent on the presence of fibronectin. There tends to be an ordered sequence of matrix deposition. Fibronectin is deposited first followed by collagen 3 and then collagen 1. Granulation tissue primarily contains type 3 collagen.

How well did you know this?
1
Not at all
2
3
4
5
Perfectly
87
Q
  1. Sebaceous glands:

A. Respond to chemical stimuli such as hormones
B. Respond to cholinergic neural activity, exclusively
C. Respond to adrenergic neural activity, exclusively
D. Respond to both adrenergic and cholinergic stimuli
E. Respond to the local release of cytokines from inflammatory cells

A

Correct choice: A. Respond to chemical stimuli such as hormones

Sebaceous glands are androgen-responsive holocrine glands that enlarge at puberty. Meibomian glands of the eyelids are modified sebaceous glands. Sebaceous glandsare found everywhere on the skin except the palms and soles. Fordyce�s condition involves free sebaceous glands on the

vermillion border of the lips and on the buccal mucosa. Eccrine glands are thermoregulatory structures that respond to cholinergic stimulation.

How well did you know this?
1
Not at all
2
3
4
5
Perfectly
88
Q
  1. The function of glycosaminoglycans/proteoglycans in the dermis is:

A. Regulate water-binding capacity
B. Interact with dermal dendrocytes
C. Facilitate COLVII binding to the anchoring plaques
D. Facilitate mast cell degranulation

A

Correct choice: A. Regulate water-binding capacity

The function of glycosaminoglycans/proteoglycans in the dermis is to regulate water-binding capacity. The other listed options are not functions of glycosaminoglycans / proteoglycans.

How well did you know this?
1
Not at all
2
3
4
5
Perfectly
89
Q
  1. Which one of the following is responsible for maintaining a barrier to water loss in the stratum corneum?

A. Involucrin
B. Filaggrin
C. Loricrin
D. Transglutaminase
E. Odland bodies

A

Correct choice: E. Odland bodies

Odland bodies, also known as lamellar granules, keratinosomes, and membrane-coating granules, are small organelles that are discharged from granular cells into the intracellular space of the granular layer of the epidermis. These bodies have two known functions: they mediate stratum corneum cell cohesion and they form a barrier to water loss. Odland bodies are round to oval, measure approximately 300 to 500 nm in diameter, and possess a trilaminar membrane and a laminated interior. They contain neutral sugars linked to lipids and/or proteins, hydrolytic enzymes, and free sterols. Filaggrin is a breakdown product of filaggrin precursor, acomponent of

keratohyaline granules, which aggregates with keratin filaments and acts as a “glue” for keratin filaments. Involucrin is a cysteine-rich protein synthesized in the cytoplasm of spinous cells. The enzyme, transglutaminase, cross-links involucrin in the granular layer forming an insoluble cell boundary that is resistant to denaturing and reducing chemicals. Loricrin, is a highly insoluble sulfur- and glycine/serine-rich protein, which is the major protein comprising the cornified cell envelope.

How well did you know this?
1
Not at all
2
3
4
5
Perfectly
90
Q
  1. During hair follicle development, the WNT signaling pathway is one of the earliest molecular pathways involved in hair follicle initiation. What is the downstream mediator of WNT signaling?

A. Smoothened
B. Beta-catenin
C. Keratin 16
D. p53
E. HLA-B27

A

Correct choice: B. Beta-catenin

Beta-catenin is the downstream mediator of WNT signaling. Through a series of signals, WNT proteins inhibit the degradation of beta-catenin in the cytoplasm. After being translocated to the nucleus, beta-catenin then forms a complex with LEF/TCF transcription factors, which ultimately results in expression of downstream genes. Activation of this pathway is necessary for epithelium to have the potential to develop a hair follicle.

How well did you know this?
1
Not at all
2
3
4
5
Perfectly
91
Q
  1. Which sebaceous gland is located on the buccal mucosa and vermi lion border of the lips?

A. Montgomery’s tubercles
B. Tyson’s glands
C. Zeis glands
D. Meibomian glands
E. Fordyce’s spots

A

Correct choice: E. Fordyce’s spots

Fordyce’s spots are located on the buccal mucosa and vermilion border of the lips. Montgomery’s tubercles are present on the areola, Tyson’s glands on the labia minora and glans, meibomian and Zeis glands are present on the eyelids.

How well did you know this?
1
Not at all
2
3
4
5
Perfectly
92
Q
  1. What structure delineates the anatomic region between the nail
    bed and the distal groove, where the nail plate detaches of the distal portion of the digit?

A. Nail matrix
B. Proximal nail fold
C. Lunula
D. Eponychium
E. Hyponychium

A

Correct choice: E. Hyponychium

The hyponychium is the structure that delineates the anatomic region between the nail bed and the distal groove, where the nail plate detaches of the distal portion of the digit. The corneal layer of the hyponychium accumulates in part under the free margin of the nail plate. In most cases, the hyponychium is covered by the distal nail plate, however, it may become visible in the case of nail biters.

How well did you know this?
1
Not at all
2
3
4
5
Perfectly
93
Q
  1. Which of the following stains would you expect to be positive in a normal eccrine unit?

A. S-100
B. Prussian blue
C. Giemsa
D. Verhoeff von Gieson
E. Steiner

A

Correct choice: A. S-100

Eccrine glands stain S-100 and CEA positive. The remaining stains would not be expected to stain normal eccrine sweat glands. Prussian blue (Perl’s) is an iron stain which stains iron or hemosiderin bright blue. Giemsa stains mast cell granules purple (heparin in the granules) and can also be useful

in staining in Leishmaniasis. Verhoeff von Gieson is a stain for elastic tissue that stains blue-black. Steiner stain is a silver stin for spirochetes similar to a Warthin Starry or Dieterle stain.

94
Q
  1. The hair follicle is the site of production or conversion of which of the following hormones?

A. All the options are correct
B. dihydrotestosterone (DHT)
C. prolactin
D. adrenocorticotropin hormone (ACTH)
E. alpha-melanocyte stimulating hormone (MSH)Question 66 Explanation

A

Correct choice: A. All the options are correct

The hair follicle converts gonadal or aderenal testosterone to dihydrotestosterone locally via 5-alpha reductase. It also cleaves proopiomelanocortin to ACTH, alpha- MSH, and B-endorphin via prohormone convertase. It also secretes corticotropin releasing hormone (CRH) as well as prolactin.

95
Q
  1. Which of the following statements about glomus cells is correct?

A. Tumors of glomus cells are most commonly found on the tongue
B. Are of neural origin
C. Allow rapid shunting of blood from the arterioles to venules, bypassing capillaries
D. Tumors composed of glomus cells are asymptomatic
E. Tumors composed of glomus cells are often malignant

A

Correct choice: C. Allow rapid shunting of blood from the arterioles to venules, bypassing capillaries.

Tumors composed of glomus cells are usually PAINFUL, not asymptomatic. They are derived from Suquet-Hoyer canals and allow rapid shunting of blood from the arterioles to venules. Glomus cells are part of glomus tumors which are most often solitary, purple dermal nodules on the extremities. Most often, they are seen on the fingers and toes. They are usually painful and rarely malignant. As opposed to a glomus tumor, glomangiomas are usually painless. They also most often occur on extremities, but can also occur on the trunk.

96
Q
  1. Where are glomus cells derived from?

A. Mesenchyme
B. Bone marrow
C. Susquet-Hoyer canal
D. Skeletal muscle
E. Dermis

A

Correct choice: C. Susquet-Hoyer canal

Glomus cells are vascular smooth muscle cells derived from the arterial portion of the glomus body, or the Sucquet-Hoyer canal, which is an arteriovenous shunt in the dermis that contributes to temperature regulation. Glomus are found primarily on the palms and soles and function to allow the rapid shunting of blood from the arteiolesto venules. Disease processes involving glomus cells include glomus tumor and glomangioma.

97
Q
  1. Which of the following cytokines is secreted by adipocytes?

A. IL-2
B. IL-4
C. IL-6
D. IL-10
E. IL-12

A

Correct choice: C. IL-6

Adipocytes secrete tumor necrosis factor (TNF)α, interleukin-6 (IL-6), leptin, adiponectin, angiotensinogen, and resistin. Specifically it has been shown that IL-6 mRNA levels in human subjects are significantly greater in adipose tissue than in other tissues known to express IL-6. It is thought that circulating IL-6 levels may be higher in obese patients, contributing to the development and progression of vascular disease in this patient population.

98
Q
  1. Dimple sign refers to:

A. When an intact epidermis shears away from the underlying dermis, leaving a moist surface
B. Spreading bulla phenomenon with pressure on an intact bulla
C. Swollen, itchy and or red after stroking the skin
D. Central depression within a lesion when squeezed along its margins
E. Disappearance of color when the lesion is pressed

A

Correct choice: D. Central depression within a lesion when squeezed along its margins

Central depression within a lesion when squeezed is referred to as the dimpling sign and is seen in dermatofibromas. Nikolsky sign can be seen when an intact epidermis shears away from the underlying dermis, leaving a moist surface (seen in pemphigus vulgaris, staphylococcus scalded skin syndrome (SSSS), and toxic epidermal necrosis). Spreading bulla phenomenon with pressure on an intact bulla is referred to as Asboe-Hansen sign, commonly seen with pemphigus vulgaris. Swollen, itchy and or red after stroking the skin is referred to Darier’s sign and can be seen in systemic mastocytosis or urticaria pigmentosa. Disappearance of color or blanching when the lesion is pressed is commonly found on vascular lesions.

99
Q
  1. Acid keratins are expressed on which of the following chromosomes?

A. 12
B. 17
C. 9
D. 16
E. 3

A

Correct choice: B. 17

Acidic keratins (K9-20) are expressed from chromosome 17. Basic keratins (K1-8) are on chromosome 12. The other listed chromosomes are not involved in keratins.

100
Q
  1. Which hormone is homologous to alpha-MSH (melanocyte stimulating hormone)?

A. Insulin

B. Human growth factor
C. Prolactin
D. Thyroid stimulating hormone
E. Adrenocorticotropic hormone

A

Correct choice: E. Adrenocorticotropic hormone

Both alpha-MSH and ACTH are cleavage products of proopiomelanocortin (POMC).

101
Q
  1. What is the average duration of the telogen cycle in terminal scalp hair?

A. 2-6 years
B. 2-3 weeks
C. 3 months
D. 6 months
E. 9 months

A

Correct choice: C. 3 months

The average duration of the telogen phase of the hair cycle is 3 months; this feature explains why telogen effluvium is typically observed 3 months following a traumatic event or serious illness. The average duration of the anagen phase of the hair cycle is 2-6 years, whereas that of the catagen cycle is 2-3 weeks.

102
Q
  1. Which sebaceous gland is located on the areola?

A. Montgomery’€™s tubercles
B. Tyson’s glands
C. Zeis glands
D. Meibomian glands
E. Fordyce’s spots

A

Correct choice: A. Montgomery’€™s tubercles

Montgomery’€™s tubercles are sebaceous glands not associated with hair follicles present on the areola. Tyson’€™s glands on the labia minora and glans, meibomina and Zeis glands are present on the eyelids , and Fordyce’€™s spots are located on the buccal mucosa.

103
Q
  1. Embryologically, epidermal stratification occurs at approximately what estimated gestational age?

A. 4 weeks
B. 8 weeks
C. 12 weeks
D. 16 weeks
E. 20 weeks

A

Correct choice: B. 8 weeks

Epidermal stratification begins at about 8 weeks estimated gestational age and is completed by the 2nd trimester. Epideral stratification occurs when an ‘intermediate layer’ develops between the epidermal basal cells and the overlying layer of periderm cells. The periderm is an embryonic structure that covers the primitive epidermis until keratinization begins, at which point the periderm sloughs off and contributes to the protective covering of the newborn, the vernix caseosa. This intermediate layer is highly proliferative, such that by 24-25 weeks estimated gestaional age, the epidermis consists of 4 or 5 layers, in addition to the degenerating eriderm.

104
Q
  1. Cutaneous warts in renal transplant recipients show increased expression of which keratin?

A. K2
B. K5
C. K9
D. K13
E. K16

A

Correct choice: D. K13

Keratins are divided into acidic (type I) and basic (type II) subtypes. Keratin 13 is a type I keratin which is expressed suprabasally in adult epithelia and is associated with terminal differentiation. Renal transplant recipients have an increased risk of developing skin cancers and multiple warts. Factors contributing to this increased risk include human papillomavirus infection, immunosuppressive therapy, and exposure to ultraviolet radiation. Increased expression of K13 has been demonstrated in high-risk papillomas in this patient population.

105
Q
  1. The finding on DIF that reflects binding of the Ro and La antigens in subacute cutaneous lupus erythematosus is:

A. Granular fluorescence throughout the cytoplasm and nucleus of basal keratinocytes
B. Cytoid bodies
C. Immune deposits along the DE junction
D. Granular deposits along the basement membrane
E. A “chicken-wire” pattern within the epidermis

A

Correct choice: A. Granular fluorescence throughout the cytoplasm and nucleus of basal keratinocytes.

Granular fluorescence throughout the cytoplasm and nucleus of basal keratinocytes reflect the binding of Ro and La antigens and is unique to SCLE. Cytoid bodies and Immune deposits along dermal-epidermal junction are seen in both DLE and SCLE.
Granular deposits along the basement membrane are seen in dermatitis herpetiformis and a chicken-wire pattern of staining is seen in pemphigus vulgaris.

106
Q
  1. At any one time, the approximate proportion of hair follicles in anagen is:

A. 40%
B. 60%
C. 85%
D. 95%
E. 15%

A

Correct choice: C. 85%

Most hair follicles are in anagen, and thus most hair follicles involve growing hair. The longer a hair follicle is anagen, the longer the hair can grow in length. Hairs of the scalp grow approximately
0.4 mm per day, and thus the date of your next hair cut can be accurately calculated.

107
Q
  1. Retinoids upregulate transcription of which types of collagen?

A. 1 and 3
B. 1 and 4
C. 1 and 7
D. 3 and 7
E. 4 and 7

A

Correct choice: A. 1 and 3

Retinoids upregulate the transcription of collagens one and thereby strengthening the dermis.

108
Q
  1. People with darker skin show:

A. Smaller, more concentrated melanosomes
B. A more rapid degradation of melanosomes
C. A lessened production of melanosomes within melanocytes
D. A higher degree of dispersion of melanosomes in keratinocytes
E. A grouping of melanosomes with a low degree of melanization

A

Correct choice: D. A higher degree of dispersion of melanosomes in keratinocytes

More darkly pigmented races show a greater production of melanosomes in the melanocyte, melanosomes with a higher degree of melanization, larger melanosomes, a higher degree of dispersion of melanosomes in the keratinocytes, and a slower rate of melanosome degradation.

109
Q
  1. What is the location of the unbound corticosteroid receptor?

A. Cytoplasm
B. Nucleus
C. Mitochondria
D. Plasma membrane
E. golgi apparatus

A

Correct choice: A. Cytoplasm

Both androgen and corticosteroid receptors localize to the cytoplasm. Estrogen receptors are found in the nucleus. Progesterone receptors are distributed in both the nucleus and the cytoplasm.

110
Q
  1. Which of the following diseases has decreased or absent lamellar granules?

A. Flegel’s
B. Psoriasis
C. Lamellar ichthyosis
D. Epidermolytic hyperkeratosis
E. Pemphigus vulgaris

A

Correct choice: A. Flegel’s

Flegel’s disease has decreased or absent lamellar granules. AKA hyperkeratosis lenticularis perstans, this is a rare, possibly AD disorder with multiple disc-like keratotic papules predominenty on the distal extremities and feet of older individuals (Bolognia, p 1714).

111
Q
  1. Elastic fibers are present in the dermis and are responsible for providing tissue resiliency. They are comprised of elastin as well as microfibrillar proteins including:

A. Fibrillins and fibulins
B. Hyaluronic acid
C. Collagen II
D. Laminin 5 and BPAG2
E. Desmoplakin and plakoglobin

A

Correct choice: A. Fibrillins and fibulins

Elastic fibers are comprised of elastin as well as microfibrillar proteins including fibrillins and fibulins. Hyaluronic acid is a glycosaminoglycan found in the extracellular matrix of the dermis. Collagen II is the predominant collagen found in cartilage. Laminin 5 and BPAG2 are important anchoring filaments in the basement membrane zone. Desmoplakin and plakoglobulin are proteins found in the anchoring plaques of desmosomes in the epidermis.

112
Q
  1. Which of the following amino acids are typically found in elastic fibers?

A. Desmosine
B. Isoleucine
C. Proline
D. Hydroxyproline
E. Glycine

A

Correct choice: A. Desmosine

Desmosine and isodesmosine are typical amino acids found in elastic fibers. Glycine, proline and hydroxyproline are found as components of collagen. Isoleucine is not specific for elastin fibers.

113
Q
  1. At what estimated gestational age are all layers of the keratinized epidermis identifiable?

A. 8 weeks
B. 12 weeks
C. 16 weeks
D. 20 weeks
E. 24 weeks

A

Correct choice: E. 24 weeks

At 24 weeks, all the layers of the mature epidermis can be identified, and the epidermis is keratinized.

114
Q
  1. Anchoring filaments originate at the hemidesmosomes and insert into the:

A. Desmosome
B. Sub basal dense plate
C. Lamina lucida
D. BPAG 180
E. Lamina densa

A

Correct choice: E. Lamina densa

Anchoring filaments (smaller than anchoring fibrils) stretch from the plasma
membrane through the subbasal dense plaque and the lamina lucida to the lamina densa.

115
Q
  1. Which of the following make up the major protein of the cornified cell envelope?

A. Loricrin
B. Involucrin
C. Envoplakin
D. Filaggrin
E. Laminin V

A

Correct choice: A. Loricrin

Loricrin is the major protein component of the cornified cell envelope (CE). Involucrin is cross- linked by transglutaminase in the granular layer to form an insoluble cell boundary. Envoplakin may link the CE to desmosomes and to keratin filaments. Filaggrin is thought to promote aggregation and disulfide bonding of keratin filaments in CE. It is degraded into urocanic acid and pyrrolidone carboxylic acid. Both of which hydrate the stratum corneum and block UV radiation. Laminin V is found in the basement membrane and is not involved in the formation of the cornified cell envelope.

116
Q
  1. Glomus cells are primarily found:

A. on hands/feet

B. on the trunk
C. on the lateral thighs
D. on the genital skin
E. on the faceQuestion 88 Explanation

A

Correct choice: A. on hands/feet

Glomus cells are derived from Susquet-Hoyer canals which function to shunt blood from the arterioles to venules and are primarily found on hands and feet. There are two types of glomus tumors, solitary and multiple. The solitary types tend to have paroxysmal pain which can be extreme. Multiple glomus tumors can be a autosomally dominant trait with incomplete penetrance and are less likely to be painful. Both have a predominance on the distal extremities, especially sub- ungual. Two tests are helpful in diagnosing glomus tumors: Hildreth sign - disappearance of pain following application of a tourniquet proximally. Love test - eliciting pain by applying pressure to a precise area with the tip of a pencil.

117
Q
  1. Which cadherin is responsible for adhesion of Langerhan cells to the epidermis?

A. E-Cadherin
B. P-Cadherin
C. N-Cadherin
D. Desmoglein
E. Desmocollin

A

Correct choice: A. E-cadherin
E-cadherin is responsible for the adhesion of Langerhan cells to the epidermis. There are two major subclasses of cadherins which mediate cell adhesion and play a fundamental role in normal development, classic (E-,P-,N-cadherin) and desmosomal (desmoglein and desmocollin). They depend on calcium for their function.

118
Q
  1. A 78 year-old man is diagnosed with cicatricial pemphigoid. Which of the following antigens being implicated in his disease should trigger a work-up for malignancy?

A. Bpag2

B. Laminin 5
C. Laminin 6
D. Beta-4 Integrin
E. Type VII collagen

A

Correct choice: B. Laminin 5

Involvement of Laminin 5 would raise the suspicion for malignancy when found. ~30- 40% of cases may have associated malignancy. Patients with antiepiligrin (laminin 5)CP indicated an increased risk of malignancy that approximates that for adults with dermatomyositis. The risk is particularly high in the first year of disease (E-medicine, www.emedicine.com/derm/topic79.htm). Reports of both lung and gastric carcinomas are seen in the literature with some feeling that gastric malignancy is most common.

119
Q
  1. What component is the major barrier in the stratum corneum?

A. Sebum
B. Squalene
C. Collagen
D. Ceramide
E. Triglycerides

A

Correct choice: D. Ceramide

The major component of lamellar granules of the keratinocytes is ceramide. These play a major role in the barrier function of the skin.

120
Q
  1. Apocrine chromhidrosis results from which of the following contents of apocrine sweat?

A. Lipofuschin
B. Squalene
C. Cholesterol
D. Fatty Acids
E. Ammonia

A

Correct choice: A. Lipofuschin

Chromhidrosis refers to the secretion of pigmented sweat, most commonly yellow, green or black. It reflects the rich lipofuschin content of apocrine sweat. Extrinsic apocrine chromhidrosis results from staining of sweat and garments by chromogenic bacteria, such as Corynebacterium spp.

121
Q
  1. What is the significance of the critical line of Auber?

A. It is the location of the insertion of the erector pili muscle
B. The bulk of the mitotic activity in the hair occurs above this line
C. The inner root sheath is formed above this line
D. It is the widest diameter of the hair bulb
E. It is where keratinization first occurs in the hair

A

Correct choice: D. It is the widest diameter of the hair bulb

The critical line of Auber is at the widest diameter fo the hair bulb. Below this line, the bulk of mitotic activity that gives rise to the hair and the inner root sheath occurs. The erector pili muscle inserts in the isthmus region of the follicle. Keratinization first occurs above this line.

122
Q
  1. Asboe-Hansen Sign refers to:

A. When an intact epidermis shears away from the underlying dermis, leaving a moist surface
B. Spreading bulla phenomenon with pressure on an intact bulla
C. Swollen, itchy and or red after stroking the skin
D. Central depression within a lesion when squeezed along its margins
E. Disappearance of color when the lesion is pressed

A

Correct choice: B. Spreading bulla phenomenon with pressure on an intact bulla

Spreading bulla phenomenon with pressure on an intact bulla is referred to as Asboe-Hansen sign, commonly seen with pemphigus vulgaris. Nikolsky sign can be seen when an intact epidermis shears away from the underlying dermis, leaving a moist surface (seen in pemphigus vulgaris,

staphylococcus scalded skin syndrome (SSSS), and toxic epidermal necrosis). Swollen, itchy and or red after stroking the skin is referred to Darier’s sign and can be seen in systemic mastocytosis or urticaria pigmentosa. Central depression within a lesion when squeezed is referred to as the dimpling sign and is seen in dermatofibromas. Disappearance of color or blanching when the lesion is pressed is commonly found on vascular lesions

123
Q
  1. Anagen effluvium is

A. Rarely seen following administration of cancer chemotherapeutic agents
B. Stimulus induces the abrupt cessation of mitotic activity in the rapidly dividing hair matrix cells
C. Occurs within 24 to 48 hours of exposure
D. Entirely irreversible
E. Patients being treated with nitrosurea agents are usually spared

A

Correct choice: B. Stimulus induces the abrupt cessation of mitotic activity in the rapidly dividing hair matrix cells

It is frequently seen following administration of cancer chemotherapeutic agents. Stimulus induces the abrupt cessation of mitotic activity in the rapidly dividing hair matrix cells. Occurs within days to weeks following exposure to the inciting agent and is entirely reversible. Frequent causes include antimetabolites, alkylating agents, miotic inhibitors, thallium, boron. Frequent causes include doxorubicin, nitrosureas and cyclophosphamide.

124
Q
  1. Sebaceous glands secrete sebum via:

A. Holocrine mechanism
B. Exocrine mechanism
C. Endocrine mechanism
D. Exostosis
E. Mecrocrine mechanism

A

Correct choice: A. Holocrine mechanism

The sebaceous lobules have basal germinative cells and central sebocytes, which gradually become more distended with lipid vacuoles until they are shed into the lumen (holocrine secretion).

125
Q
  1. Homocystinuria has abnormal crosslinking of collagen because of a mutation in:

A. Cystathione synthase
B. Type i collagen n-peptidase gene
C. Lysyl hydroxylase
D. Tenascin x
E. Lysyl oxidase

A

Correct choice: A. Cystathione synthase

Homocystinuria is caused by a mutation in cystathione synthase. The main skin findings are a malar flush, livedo reticularis and leg ulcerations. A characteristic eye finding is the downward displacement of the lens. The other options are involved in abnormalities associated with Ehlers- Danlos syndrome. Lysyl hydroxylase is deficient in Kyphoscoliosis type of EDS. Tenascin X is involves in ~3% of Classical type EDScases. Dermatosparaxis type EDS has recessive mutations in the type I collagen N-peptidase gene.

126
Q
  1. During a salt split skin test, if the location of the deposition if found
    on the roof€™ (lamina lucida), which of the following could be a diagnosis?

A. Bullous pemphigoid
B. Anti-epiligrin cicatricial pemphigoid
C. Epidermolysis bullosa acquisita
D. Bullous eruption of lupus erythematosus
E. Bullous eruption of tinea

A

Correct choice: A. Bullous pemphigoid

Bullous pemphigoid is one of the many bullous diseases that may be found on the ‘roof’ of the blister, the others include pemphigoid gestationis, linear IgA bullous dermatosis, cicatricial pemphigoid. The other answer (except for bullous tinea, in which a salt split

would not be performed) are those found on the ‘floor’ of the blister (lamina densa).

127
Q
  1. Which of the following polypeptides is found in the lamina lucida?

A. Plakoglobin
B. Desmoplakin
C. Keratocalmin
D. Demoyokin
E. Laminin 5

A

Correct choice: E. Laminin 5

Laminin-5 is a basement membrane extracellular matrix protein that mediates attachment substrate for both adhesion and migration in a wide variety of cell types, including epithelial cells, fibroblasts, neurons and leukocytes and is a preferred adhesion substrate for epithelial cells (Koshikawa et al., 2001). The remaining listed items are part of the desmosomal plaque in the epidermis.

128
Q
  1. When do melanocytes begin to synthesize melanin?

A. 2nd month of gestation
B. 3rd month of gestation
C. 4th month of gestation
D. 5th month of gestation
E. 6th month of gestation

A

Correct choice: B. 3rd month of gestation
Melanocytes begin to synthesize melanin in the 3rd month of gestation.

129
Q
  1. For the treatment of drug induced linear IgA bullous dermatosis, after stopping offensive medication, what other treatments may be used?

A. Interferon alpha
B. Interferon gamma
C. Psoralen plus uva
D. Interleukin 2
E. Dapsone

A

Correct choice: E. Dapsone

Dapsone may be used to treat patients with drug induced linear IgA bullous dermatosis which persist after the offending medication has been discontinued. The other medications listed are rare causes of drug induced linear IgA bullous dermatosis and so should not be used for treatment.

130
Q
  1. Granulation tissue primarily contains:

A. Collagen I
B. Collagen III
C. Collagen IV
D. Fibrin
E. Collagen VII

A

Correct choice: B. Collagen III

Granulation tissue begins to form four days after injury and is composed of new capillaries, macrophages, fibroblasts, blood vessels and primarily collagen type III. Formulation of granulation tissue is dependent on the presence of fibronectin and progresses through an orderly sequence of matrix deposition: fibronectin to collagen III then finally collagen I.

131
Q
  1. The antibody target in ocular cicatricial pemphigoid is also mutated in:

a. Junctional epidermolysis bullosa, Herlitz type
B. Recessive dystrophic epidermolysis bullosa

C. Junctional epidermolysis bullosa with myotonic dystrophy
D. Dominant dystrophic epidermolysis bullosa
E. Junctional epidermolysis bullosa with pyloric atresia

A

Correct choice: E. Junctional epidermolysis bullosa with pyloric atresia

JEB with pyloric atresia and ocular cicatricial pemphigoid both have mutations in Beta4integrin. Herlitz type JEB is due to laminin 5. Recessive and dominant dystrophic EB is due to COL17A1 (type VII collagen). JEB with myotonic dystrophy is due to plectin.

132
Q
  1. Which of the following is true regarding the development of hair follicles?

A. The first primordial hair follicles form at 15 weeks gestation
B. The first hair follicles form on the scalp and eyelashes
C. Follicles develop in a cephalad to caudal direction
D. New follicles develop during the first 3 months postpartum
E. The eyebrows develop late in gestation

A

Correct choice: C. Follicles develop in a cephalad to caudal direction

The first primordial hair follicles form at 9 weeks gestation on the eyebrows, upper lip, and chin. The remaining follicles develop at 4-5 months in a cephalad to caudal direction. New follicles cannot develop in adult skin.

133
Q
  1. What does a western blot identify?

A. Protein
B. RNA
C. DNA
D. Genes
E. Cell membranes

A

Correct choice: A. Protein

A northern blot identifies RNA and a southern blot identifies DNA.

134
Q
  1. Sneddon Wilkinson disease is caused by a defect in what antigen?

A. Desmocollin 1
B. Desmoglein 1
C. Desmogelin 3
D. Bpag1
E. Bpag2

A

Correct choice: A. Desmocollin 1

Sneddon- Wilkinson disease, also known as sub-corneal pustular dermatosis is due to a defect in desmocollin 1. Desmoglein 1 defect is seen in pemphigus foliaceous and staph scalded skin syndrome. Desmoglein 3 defect is seen in pemphigus vulgaris and intraepidermal neutrophilic IgA dermatosis. BPAg1 is seen in paraneoplastic pemphigus and bullous pemphigoid. BPAg2 is seen in bullous pemphigoid, cicatricial mucous membrane pemphigoid and linear IgA disease.

135
Q
  1. Apocrine glands:

A. Demonstrate holocrine secretion
B. Demonstrate decapitation secretion
C. Are fully functional at birth
D. Are diffusely distributed on the body
E. Are thermoregulatory

A

Correct choice: B. Demonstrate decapitation secretion

Apocrine glands show decapitation secretion. Like eccrine glands, apocrine glands are composed of three segments, the intraepidermal duct, the intradermal duct, and the secretory portion. The duct of the apocrine gland usually leads to a pilosebaceous follicle above the entrance of the sebaceous duct. Apocrine glands are found in the axillae, anogenital region, external ear canal (ceruminous glands), in the eyelids (Moll’s glands), and in the breast (mammary glands). Apocrine glans are functional only at puberty. Their initial secretion is odorless.

136
Q
  1. Which protein is the largest component of the cornified cell envelope?

A. Keratin
B. Involucrin
C. Profilaggrin
D. Loricrin
E. Ceramide

A

Correct choice: D. Loricrin

The cornified cell envelope is a durable, protein-lipid polymer that eventually acts as a mechanical and chemical barrier on the exterior of cornified cells. In the upper spinous layer, keratohyaline granules release profilagrin and loricrin. Profilaggrin is cleaved to filaggrin and subsequently aggregates keratin filaments. Loricrin is the major protein component of the cornified cell envelope and is bound to the cell membrane by transglutaminases (in addition to other structural proteins like involucrin, keratins, elafin, cystatin A and desmosomal peptides). This forms the
highly insoluble proteinaceous component of the cornified cell envelope.

137
Q
  1. Acid keratins (K10-20) are expressed on which of the following chromosomes?

A. 17
B. 18
C. 19
D. 10
E. 12

A

Correct choice: A. 17

Acid keratins are expressed on chromosome 17 and basic keratins on chromosome 12. The other listed options do not have active keratin functions.

138
Q
  1. Defects in what kind of structural protein lead to pyloric atresia associated with junctional epidermolysis bullosa:

A. Collagen
B. Elastin
C. Loricrin
D. Integrin
E. Plectin

A

Correct choice: D. Integrin

Junctional epidermolysis bullosa with pyloric atresia involves a defect in the b4 subunit of the a6b4 integrin. The expression of this protein is limited to the basal layer of the epidermis. This integrin is a transmembrane protein that coordinates a link between the intermediate filaments (keratins) and the extracellular matrix of the basement membrane. The b4 domain mediates an interaction with both plectin and BP180; its absence prevents hemidesmosomal assembly.

139
Q
  1. Which of the following cells are required for wound healing?

A. Neutrophil
B. Macrophage
C. Eosinophil
D. Langerhans cell
E. Lymphocyte

A

Correct choice: B. Macrophage

The macrophage is required for wound healing. The macrophages debride tissue, secrete collagenase and stimulate expression of FGF, IL-1, TGF-beta, PDGF and TGF-alpha thus facilitating transition from inflammation to repair.

140
Q
  1. Which of the following options is characteristic of integrins?

A. Defects in the alpha-6 domain of integrin result in epidermolysis bullosa simplex with muscular dystrophy

B. Alpha-6-beta-4 integrin is found at sites where desmogleins attach
C. Expression is seen in all layers of the epidermis
D. The extracellular alpha6 domain binds collagen 7
E. These proteins coordinate linkage between intermediate filaments and extracellular matrix of the basement membrane

A

Correct choice: E. These proteins coordinate linkage between intermediate filaments and extracellular matrix of the basement membrane

Defects in the BETA-4 (not alpha-6) domain of integrin result in junctional epidermolysis bullosa with pyloric atresia. Its expression is seen in the basal cell layer and binds to laminins.

141
Q
  1. Desmoglein 1 is the antigen in which of the following autoimmune diseases of the skin:

A. Pemphigus foliaceus
B. Bullous impetigo
C. Dermatitis herpetiformis
D. Bullous pemphigoid
E. Pemphigoid gestationis

A

Correct choice: A. Pemphigus foliaceus

The antigen implicated in pemphigus foliaceus is Desmoglein 1. Desmoglein 1 is targeted in bullous impetigo, but this is an infectious condition, not an autoimmune disease. The antigen in dermatitis herpetiformis is transglutaminase 3. The antigens for both bullous pemphigoid and pemphigoid gestationis are BPAG1 and BPAG2.

142
Q
  1. Which of the following keratins would most likely be expressed in the nail bed?

A. K6a/16
B. K6b/17
C. K1/9
D. K2e/10
E. K4/13

A

Correct choice: B. K6b/17

Keratins 6b & 17 are expressed in the nail bed. K6a/16 is expressed in the outer root sheath and in hyperproliferative keratinocytes, 1/9 in palmoplantar suprabasalar keratinocytes, 2e/10 in the upper spinous and granular cell layers, and 4/13 are expressed in mucosal epithelium.

143
Q
  1. Which of the following abnormalities would be present in a patient with Harlequin ichthyosis?

A. Lamellar granules are uniformly absent
B. Keratohyaline granules are normal in all types of harlequin fetus
C. Demoyokin mutation
D. Band 6 protein is absent
E. Plakoglobin is abnormal

A

Correct choice: A. Lamellar granules are uniformly absent

In Harlequin ichthyosis, the lamellar granules are uniformly abnormal or absent. On electron microscopy, there is no evidence of the lipid lamellae which form due to lamellar granule discharge into the region between the granular and cornified cells (Fitz v6, p493). This is suggestive of a primary defect in lipid synthesis and protein dephosphorylation resulting in faulty lamellar body formation and secretion (Bolognia, ch. 57). Harlequin fetus/ichthyosis can be divided into three types based on the microscopic appearance of the keratohyaline granules. Type I - normal, Type II - too small to be seen by light microscopy, Type III - absent keratohyaline granules. Demoyokin, band-6 protein (a plaque component) and plakoglobin are desmosomal proteins and not involved in Harlequin ichthyosis. Recently, the gene defect was localized a defect in the ABCA12 protein.

144
Q
  1. Nikolsky sign can be seen when:

A. When an intact epidermis shears away from the underlying dermis, leaving a moist surface
B. Spreading bulla phenomenon with pressure on an intact bulla
C. Swollen, itchy and or red after stroking the skin
D. Central depression within a lesion when squeezed along its margins
E. Disappearance of color when the lesion is pressed

A

Correct choice: A. When an intact epidermis shears away from the underlying dermis, leaving a moist surface

Nikolsky sign can be seen when an intact epidermis shears away from the underlying dermis, leaving a moist surface (seen in pemphigus vulgaris, staphylococcus scalded skin syndrome (SSSS), and toxic epidermal necrosis). Spreading bulla phenomenon with pressure on an intact bulla is referred to as Asboe-Hansen sign, commonly seen with pemphigus vulgaris. Swollen, itchy and or red after stroking the skin is referred to Darier’s sign and can be seen in systemic mastocytosis or urticaria pigmentosa. Central depression within a lesion when squeezed is referred to as the dimpling sign and is seen in dermatofibromas. Disappearance of color or blanching when the lesion is pressed is commonly found on vascular lesions

145
Q
  1. Acral melanomas are particularly concerning because they:

A. Are diagnosed at a later stage
B. Invade perineurally
C. Cannot be resected
D. Metastasize frequently even at shallow breslow depth
E. Do not respond to ipilimumab

A

Correct choice: A. Are diagnosed at a later stage

Acral melanoma is problematic because it is diagnosed at a later stage. It accounts for 5-10% of all melanomas.

146
Q
  1. Keratinocytes in the basal layer of the epidermis attach to the basement membrane zone at hemidesmosomes via what intermediate filament molecules?

A. Keratins 1 and 10
B. Keratins 5 and 14
C. Desmogleins 1 and 3
D. Desmoplakin and desmoglobin
E. Bpag1 and bpag2

A

Correct choice: B. Keratins 5 and 14

Keratins 5 and 14 are intermediate filaments found in the basal layer of the epidermis that play a role in attaching basal layer keratinocytes to the basement membrane zone. Keratins 1 and 10 are found in more superficial layers of the epidermis. Desmogleins, desmoplakin, and desmoglobin are all components of desmosomes which attach keratinocytes to other surrounding keratinocytes. BPAG1 and BPAG2 are basement membrane proteins.

147
Q
  1. The proteins that make up the cornified cell envelope are synthesized in the:

A. Stratum corneum
B. Stratum basale
C. Melanocytes
D. Stratum granulosum
E. Langerhans cells

A

Correct choice: D. Stratum granulosum

The proteins that make up the cornified cell envelope are synthesized in the spinousand granular cell layer. The stratum basale is not involved in production of these proteins and there is no protein synthesis in the stratum corneum. Melanocytes and Langerhans cells are components of the epidermis but are not involved in this process.

148
Q
  1. Which keratins are expressed in the stratum germinativum and are present but not made de novo in the stratum spinosum?

A. K4, 13
B. K1,10
C. K2e, 10
D. K3,12
E. K5,14

A

Correct choice: E. K5,14

K5 and K14 are expressed in the stratum germinativum (basal layer) and are
defective in epidermolysis bullosa simplex. They are still present in the stratum spinosum but are not made de novo in this layer. K4 and 13 are found in mucosal epithelium. K1 and 10 are expressed in suprabasal keratinocytes, K2e and 10 in the upper spinous and granular layers, and K3 and K12 in the cornea.

149
Q
  1. Fibrofolliculomas are associated with:

A. Birt-hogg-dube syndrome
B. Cowdens
C. Cutis laxa
D. Marfans
E. Brook spiegler

A

Correct choice: A. Birt-Hogg-Dube syndrome

Fibrofolliculomas are associated with Birt-Hogg-Dube Syndrome and not in the other answer choices.

150
Q
  1. Which of the following skin conditions involves elastin?

A. Rothmund-thompson
B. Epidermolytic hyperkeratosis
C. Pseudoxanthoma elasticum
D. Progeria
E. Classic type ehlers-danlos syndrome

A

Correct choice: C. Pseudoxanthoma elasticum

There are many genodermatoses that involve elastin. Pseudoxanthoma elasticum shows increased glycosaminoglycans on elastic fibers, calcium deposition and accumulation of fragmented and calcified elastic fibers. Cutis laxa shows decreased desmosine and lysyl oxidase. In Marfan sydrome there is decreased fibrillin I and fragmentation of elastic fibers. Buschke-Ollendorf syndroms shows

increased desmosine and increased amount of thickend elastic fibers. Other elastin disease are congenital contractural arachnodactyly from a mutation in fibrillin II and anetoderma
which shows decreased desmosine and loss/fragmentation of elastic fibers. Classic type Ehlers- Danlos syndrome is caused by a defect in collagen V and not in elastin. Progeria is caused by a defect in lamin A. EHK is caused by a defect

151
Q
  1. Which mechanoreceptor found in hair bearing areas sense deep touch and vibration?

A. Merkel cell
B. Meissner corpuscle
C. Vater-pacini corpuscle
D. Krase end-bulb
E. Free nerve ending

A

Correct choice: C. Vater-pacini corpuscle

Vater-Pacini corpuscles sense deep touch and vibration. Merkel cells are slow adapting type I mechanoreceptors found among basal keratinocytes. Meissner corpuscles are found in the dermal papilla, especially in the palms and soles. Krase end-bulbs are mucocutaneous end-organs found on the glans penis, prepuce, clitoris, labia minora, and vermillion border of the lip.

152
Q
  1. A patient presents with painless firm nodules. Pathology reveals no epidermal involvement. The patient is ultimately diagnosed with sarcoid. What is this type of involvement called?

A. Darier-roussy disease
B. Lofgren’s syndrome
C. Heerfordt’s syndrome
D. Lupus pernio
E. Kveim-siltzbach disease

A

Correct choice: A. Darier-roussy disease

Darier Roussy disease refers to subcutaneous sarcoid. Lofgren’s syndrome is sarcoid presenting as erythema nodosum, hilar lymphadenopathy, fever, migrating polyarthritis, and acute iritis. Heerfordt’s is parotid gland enlargement, uveitis, fever and cranial nerve palsies. Kveim- Siltzbach antigen is used rarely in the US but can aid in the diagnosis of sarcoid.

153
Q
  1. Which keratins are upregulated in hyperproliferative disease such as psoriasis?

A. Keratins 1 and 10
B. Keratins 2e and 10
C. Keratins 5 and 14
D. Keratins 6 and 16
E. Keratins 8 and 18

A

Correct choice: D. Keratins 6 and 16

Keratins 6 and 16 are upregulated in hyperproliferative keratinocytes, outer root sheath, and oral epithelium. A defect in these keratins may result in pachyonychia congenita.

154
Q
  1. Which of the following is a member of the armadillo family of linking proteins?

A. E-cadherin
B. Periplakin
C. Envoplakin
D. Desmocollin
E. Plakoglobin

A

Correct choice: E. Plakoglobin

Plakoglobin is an example of an armadillo protein, which links the cytoskeleton associated linking proteins (such as plakins in the case of intermediate filaments and alpha-catenin in the case of actin) to the transmembrane adhesion molecules, termed cadherins. E-cadherin and desmocollin are examples of cadherins. Periplakin and envoplakin are examples of plakin linking proteins.

155
Q

155 -A specific marker of Merkel cells is:

A. Cytokeratin 10
B. Cytokeratin 15
C. Cytokeratin 20
D. Loricrin
E. Envoplakin

A

Correct choice: C. Cytokeratin 20

Cytokeratin 20 is a specific marker for the Merkel cell. Merkel cells are
mechanoreceptors located at body sites requiring high tactile sensitivity. Keratinocyte deformation results in a secretion of chemokines by Merkel cells, which make synaptic connection with neurons.

156
Q
  1. In the epidermis, the cell most responsible for antigen detection and processing is the:

A. Keratinocytes
B. Merkel cell
C. Melanocyte
D. Langerhans cell
E. Cd4+ t cell

A

Correct choice: D. Langerhans cell

The Langerhans cell is a bone narrow-derived, antigen-presenting cell found in all layers of the epidermis, oral mucosa, esophagus, and vagina. Langerhans cells ingest and process antigens, mature, migrate to a local lymph node, and then present the antigen to a na�ve (or resting) T cell, activating that T cell. The Langerhans cell is central to the pathogenesis of atopic dermatitis, psoriasis, allergic contact dermatitis, and certain infections, such as Leishmaniasis.

157
Q
  1. In which of the following locations would you be least likely to identify melanocytes?

A. Stria vascularis of the ear

B. Iris
C. Leptomeninges
D. Retina
E. Pericardium

A

Correct choice:E. Pericardium

Melanocytes are found in the stria vascularis of the ear, iris, leptomeninges and retina. There are no normal populations of melanocytes found in the pericardium.

158
Q
  1. Darier’s sign is described as:

A. When an intact epidermis shears away from the underlying dermis, leaving a moist surface
B. Spreading bulla phenomenon with pressure on an intact bulla
C. Swollen, itchy and or red after stroking the skin
D. Central depression within a lesion when squeezed along its margins
E. Disappearance of color when the lesion is pressed

A

Correct choice: C. Swollen, itchy and or red after stroking the skin

Swollen, itchy and or red after stroking the skin is referred to Darier’s sign and can be seen in systemic mastocytosis or urticaria pigmentosa. Nikolsky sign can be seen when an intact epidermis shears away from the underlying dermis, leaving a moist surface (seen in pemphigus vulgaris, staphylococcus scalded skin syndrome (SSSS), and toxic epidermal necrosis). Spreading bulla phenomenon with pressure on an intact bulla is referred to as Asboe-Hansen sign, commonly seen with pemphigus vulgaris. Central depression within a lesion when squeezed is referred to as the dimpling sign and is seen in dermatofibromas. Disappearance of color or blanching when the lesion is pressed is commonly found on vascular lesions

159
Q
  1. The embryonic periderm becomes part of the :

A. Vernix caseosa
B. Stratum corneum

C. Stratum basale
D. Dermis
E. Hair follicle

A

Correct choice: A. vernix caseosa

During the second trimester of fetal development, the periderm is sloughed frommost of the skin surface revealing the underlaying epidermis. The periderm becomes part of the protective coating, vernix caseosa, together with the shed lanugo, sebum and other amniotic fluid materials. Bolognia p.38

160
Q
  1. Which of the following statements about Laminins is correct?

A. Laminins span from the plasma membrane of basal keratinocytes to the lamina lucida
B. Laminins provide little structural support in the basement membrane
C. Laminins provide signaling molecules that interact with other proteins to transmit morphogenetic information to the cell’s interior
D. Laminin 5 is also called plectin
E. Laminin 5 is the only laminin found in the basement membrane

A

Correct choice: C. Laminins provide signaling molecules that interact with other proteins to transmit morphogenetic information to the cell’s interior

Laminins do transmit signals to the cell’s interior. They span from the plasma membrane of basal keratinocytes to the lamina DENSA, not lucida. Laminin 5 (also known as epiligrin) is part of a structural network in the basement membrane that also includes laminin 6.

161
Q
  1. Which of the following elements is necessary for melanin production?

A. Copper
B. Selenium
C. Iron
D. Zinc
E. Calcium

A

Correct choice: A. Copper

Melanin is synthesized from tyrosine via the action of the enzyme tyrosinase (tyrosine to DOPA to DOPAquinone to melanin). Tyrosinase is a copper containing enzyme. The other elements are not specifically required for melanin synthesis.

162
Q
  1. Once a keratinocyte leaves the basal cell layer, the normal transit time to stratum corneum is at least:

A. 7 days
B. 14 days
C. 21 days
D. 28 days
E. 35 days

A

Correct choice: B. 14 days

Once a basal cell leaves the basal layer in humans, normal transit time
to stratum corneum is at least 14 days. Transit time through the stratum corneum to desquamation requires 14 more days.

163
Q
  1. Regarding the stratum germinativum (basale):

A. Intermediate filaments in basal cells insert into only hemidesmosomes
B. Keratins 1 and 10 are expressed
C. Not all basal cells have the potential to divide
D. Microfilaments assist in downward movement of cells
E. Plectins regulate adhesion and initiation of differentiation.

A

Correct choice: C. Not all basal cells have the potential to divide

Not all basal cells have the potential to divide. Stem cells give rise to transient amplifying cells which give rise to the epidermal keratinocytes. Intermediate filaments in basal cells insert into

desmosomes and hemidesmosomes. Keratins 5/14 are predominantly expressed. Microfilaments assist in upward movement of cells. Integrins regulate adhesion and initiation of differentiation.

164
Q
  1. Each of the following is true about the basement membrane zone except:

A. Anchoring filaments attach the basal cell membrane to the lamin lucida
B. Can be visualized on light microscopy with pas staining
C. Lamina densa is composed of type iv collagen
D. Contains laminin 1 and laminin 5
E. Anchoring fibrils are composed of type vii collagen

A

Correct choice: A. Anchoring filaments attach the basal cell membrane to the lamin lucida

The basement membrane zone is seen on staining with PAS stain. It appears as a homogenous band approximately 1 micron thick at the dermo-epidermal junction. The hemidesmosomal complex and basement membrane zone play an integral role in maintaining cellular adhesion. Anchoring filaments (primarily composed of laminin5 and BPAG2) attach the basal cell membrane to the lamina densa NOT lamina lucida.

165
Q
  1. Regarding the stratum spinosum, which of the following is correct?

A. No keratin 1/10 is present
B. New synthesis of k5/14 occurs in this layer
C. The “spines” seen on pathology are due to desmosomal connections between keratinocytes
D. This layer contains melanocytes
E. This layer contains the cornified cell membrane

A

Correct choice: C. The “spines” seen on pathology are due to desmosomal connections between keratinocytes .

The “spines” seen on pathology are due to desmosomal connections between keratinocytes, NOT hemidesmosomes. The hemidesmosomes are present in the cells at the base of the basal layer and

are part of the connection between the epidermis and basement membrane. Keratin expression continues in the spinous layer. Keratins 1/10 are synthesized, not keratins 5/14. Keratin 5/14, however, is still present. Keratohyaline granules are typically seen in the granular layer.

166
Q
  1. Choose the correct answer regarding melanin and skin color:

A. In black and brown skin the melanosomes are smaller in diameter and length
B. Facultative skin color is the amount of cutaneous melanin pigment generated according to cellular genetics
C. In white skin the melanosomes form groups within the secondary lysosomes
D. Eumelanin produces a yellow chromophore
E. The number of melanocytes increases with one exposure to UVA/visible light

A

Correct choice: C. In white skin the melanosomes form groups within the secondary lysosomes

Melanocytes of dark skin synthesize melanosomes larger than those produced in light skin. The number of melanocytes in the epidermis is the same, regardless of the person’s race or color: it is the number and size of the melanosomes or pigment granules, continuously synthesized by these melanocytes, that determine differences in skin color. The size of the melanosome is the principle factor in determining how the melanosomes will be distributed within the keratinocytes. The larger the melanosomes of dark skin are individually dispersed within the cytoplasm of keratinocytes: smaller melanosomes of light skin are packaged in membrane-Bound complexes within keratinocytes. Eumelanin is in dark oval melanosomes found in black hair

167
Q
  1. As you move upward through the epidermis toward the stratum corneum, which of the following is true?

A. Calcium increases, phospholipids decrease, sphingolipids decrease
B. Calcium increases, phospholipids increase sphingolipids increase
C. Calcium increases, phospholipids decrease, sphingolipids increase
D. Calcium decreases, phospholipids decrease, sphingolipids decrease
E. Calcium decreases, phospholipids increase sphingolipids decrease

A

Correct choice: C. calcium increases, phospholipids decrease, sphingolipids increase

Calcium increases, phospholipids decrease, sphingolipids increase. Calcium is needed for desmosome formation and enzyme activation. Phospholipid content of epidermis decreases with differentiation but neutral lipids and sphingolipids (ceramide) increase with differentiation.

168
Q
  1. Pick the correctly paired keratin with its structure:

A. K1/k10 - basal cells
B. K3/k12 - esophagus
C. K4/k13 - cornea
D. K5/k14 - suprabasal cells
E. K1/k9 - Palms and soles

A

Correct choice: E. K1/K9 - palms and soles

Keratin 1 and 10 are found in the stratum spinosum. K5 and K14 are found in the basal layer. K3 and K12 are found in the suprabasilar cells of the cornea. K4 and K13 are found in the non- cornifying cells of stratified mucosa. K1 and K9 are found in the palms and soles.

169
Q
  1. Which of the following groups of adhesion proteins are found in both the hemidesomsome- anchoring filament complexes and lamina densa?

A. Plectin
B. Heparin sulfate proteoglycan
C. Laminin 5
D. Nidogen
E. Type vii collagen

A

Correct choice: C. Laminin 5

Laminin 5 is found both in the hemidesomsome-anchoring filament complexes and lamina densa. Plectin is found in the hemidesomsome-
anchoring filament complexes.
Heparin sulfate proteoglycan is found only in the lamina densa, collagen VII in the sublamina densa, and nidogen in the lamina densa.

170
Q
  1. All of the following are consider intermediate filament except:

A. Microtubules
B. Keratins
C. Vimentin
D. Desmin
E. Peripherin

A

Correct choice: A. Microtubules

Intermediate filaments are composed of keratins, vimentin, desmin, peripherin, neurofilaments, nuclear laminins, and nestin. These are part of cytoskeletal elements.

171
Q
  1. Mutations in which of the following genes will produce red hair?

A. MC1-R
B. Agouti
C. Hairless
D. C-kit
E. Tyrosinase

A

Correct choice: A. MC1-R

MC1-R is the receptor that binds alpha-MSH, and ACTH>alpha-MSH. Defects in this receptor will produce a phenotype including red hair. Agouti is not involved in this process. Agouti describes the banding of hairs seen in some mammals such as dogs, foxes, and mice, which is due to alternating production of eumelanin and pheomelanin occurs. This has not been described in humans (Bolognia, p942). Thehairless gene is associated with alopecia totalis. C-kit mutations are associated with piebaldism and urticaria pigmentosa. Tyrosinase defects are associated with Type I Oculocutaneous albinism.

172
Q
  1. Surgery, Parturition, Fever, Kwashiokor and Hypervitaminosis A are all causes of:

A. Telogen effluvium

B. Anagen effluvium
C. Alopecia areata
D. Androgenetic alopecia
E. Both telogen and anagen effluvium

A

Correct choice: A. Telogen effluvium

All of the listed stressors can induce telogen effluvium. Other causes include traction and some drug exposures. Usually will only involve up to 50% of scalp hairs and will resolve within 2-3 months.

173
Q
  1. Which of the following glands is not under neural control?

A. Sebaceous glands
B. Apocrine glands
C. Eccrine glands
D. Salivary glands
E. Molls glands

A

Correct choice: A. Sebaceous glands

Sebum is secreted continuously on the skin under hormonal glands. It contains squalene, cholesterol, cholesterol esters, wax, and triglycerides.

174
Q
  1. Mutations in which of the following proteins results in epidermolysis bullosa simplex associated with muscular dystrophy:

A. Uncein
B. Plectin
C. A6b4 integrin
D. Laminin 5
E. Collagen type iv

A

Correct choice: B. Plectin

EBS with muscular dystrophy is due to a plectin mutation. There are many types of EBS. Anchoring filaments exist within the lamina lucida. They are primarily comprised of laminin 5 and BP180. Laminin 5 is a cross-shaped assembly of 3 classes of polypeptides, a, b, g. The anchoring filaments function as a structural network to which other proteins attach, and they function as signaling molecules that transmit morphogenetic information to transmembrane proteins of the basal cell layer (such as the integrins).
Laminin 5 is also called epiligrin and binds to the a6b4 integrin at the hemidesmosome.

175
Q
  1. Sebaceous glands secrete sebum through which of the following secretory mechanisms?

A. Holocrine
B. Merocrine
C. Apocrine
D. Holocrine and merocrine
E. Holocrine and apocrine

A

Correct choice: A. Holocrine

Sebaceous glands exhibit holocrine secretion, whereby the sebocytes disintegrate in transit to the gland center, releasing their sebum contents. Merocrine secretion refers to the formation of intracellular secretory vesicles that translocate to the apical cell surface for secretion. Apocrine secretion refers to the process whereby secretory contents are packaged using the apical cell membrane, and ‘pinched off’ to achieve secretion.

176
Q
  1. Type VII collagen in found in anchoring fibrils and also in:

A. Fetal skin
B. Bone
C. Amnion
D. Aorta

A

Correct choice: C. Amnion

Type VII collagen is present in anchoring fibrils and amnion. Fetal skin and blood vessels contain type III collagen. The aorta contains type VI collagen. Bone contains type I collagen.

177
Q
  1. Keratin filaments in basal cells insert into:

A. Desmosomes
B. Adherens junctions
C. Connexins
D. Lamellar granules
E. Odland bodies

A

Correct choice: A. Desmosomes

Keratin filaments insert into desmosomes and hemidesmosomes in the basal cell layer and into desmosomes in the layers above. Connexins do not bind keratin filaments. Odland bodies and lamellar granules are synonyms. These are a “membrane bounded organelle, specialized for the storage and secretion various substances (surfactant phospholipids, glycoproteins and acid phosphates) which are arranged in the form of tightly packed, concentric, membrane sheets or lamellae. Has some similar properties to, but is distinct from, a lysosome.” Desmosomes are multi- protein complexes that function as cell–cell adhesion structures (junctions) in epidermal cells. They also provide attachment sites for the keratin intermediate filament cytoskeleton of keratinocytes. Consequently, these junctions are critical components of a supracellular filament network that traverses the interfollicular epidermis and the epithelia of skin appendages, such as hair follicles.

178
Q
  1. Moving from internally to externally choose the correct description of the hair follicle:

A. Inner root sheath cuticle - huxley’s layer - henle’s layer - medulla - cortex- hair shaft cuticle
B. Hair shaft cuticle - cortex - medulla - henle’s layer - huxley’s layer _ inner root sheath cuticle
C. Henle’s layer - huxley’s layer - inner root sheath cuticle - hair shaft cuticle - cortex - medulla
D. Medulla _ cortex _ hair shaft cuticle _ inner root sheath cuticle _ huxley’s layer _ henle’s layer

E. Inner root sheath cuticle _ outer root sheath cuticle _ hair shaft cuticle _ cortex _ huxley’s layer _ henle’s layer

A

Correct choice: D. Medulla _ cortex _ hair shaft cuticle _ inner root sheath cuticle _ Huxley’s layer
_ Henle’s layer (As described in correct choice)

179
Q

179- upon presentation of an antigen in the skin surface, a hapten forms. The first cell to take up the hapten is:

A. B cells
B. Langerhans cells
C. Keratinocytes
D. T cells
E. Mast cell

A

Correct choice: B. Langerhans cells

Most of the contact allergens are low-molecular weight chemicals, which after penetrating into the skin, have to couple with host proteins to be able to act as full antigens. These are called haptens. Upon epicutaneous application to a naive host, Langerhans cells take up the hapten, Process it and migrate towards the regional lymph nodes, where the anitgen is presented to the naive T cells.

180
Q
  1. Merkel cells are mechanoreceptors found in areas of high-tactile sensitivity. This immunohistochemical marker is restricted to Merkel cells in
    the skin and is thus a reliable marker for these cells:

A. Keratin 7
B. Keratin 20
C. S-100
D. Factor xiiia
E. Lyve-1

A

Correct choice: B. Keratin 20

Keratin 20 is reliable immunohistochemical markers for Merkel cells as it is restricted to these cells in the skin. Keratin 7 can be used as a marker for Paget’s Disease. S- 100 is frequently used to stain neural cells and melanocytes. Factor XIIIa can be used to differentiate a dermatofibroma from dermatofibroma sarcoma pertuberans (positive in DF; negative in DFSP). LYVE-1 is a marker for lymphatics.

181
Q
  1. Which cell type is required for wound healing?

A. Macrophage
B. Lymphocyte
C. Mast cell
D. Dermal dendrocyte
E. Neutrophil

A

Correct choice: A. Macrophage

The macrophage is required for wound healing. Without macrophages, there is no healing. Macrophages debride tissue through phagocytosis and digestion of organisms, tissue debris and effete PMN’s. They secrete
collagenase and secrete growth factors that facilitate transition from inflammation to repair.

182
Q
  1. Odland bodies:

A. Contain keratins
B. Are found intercellularly in the basal cell layer
C. Are exclusively intracellular
D. Crosslink with keratins 5 and 14
E. Are secretory granules with features of lysosomes

A

Correct choice: E. Are secretory granules with features of lysosomes

Odland bodies (AKA lamellar granules) are lamellated bodies containing ceramide, are found intercellularly in upper level keratinocytes; they discharge contents into the extracellular space at the junction of the granular and horny layers, establish a barrier to water loss, and with filaggrin mediate stratum corneum adhesion. Recent studies suggest that they are a type of secretory granule with features of lysosomes. (They contain lamellar-body-derived enzymes that are important in desquamation.)

183
Q
  1. Which signaling molecule mediates the transition of hair cycling from telogen to anagen phase?

A. Foxn1
B. Fgf5
C. Sonic hedgehog
D. Dihydotestosterone
E. 5a-reductase

A

Correct choice: C. Sonic hedgehog

Sonic hedgehog (Shh), a signaling molecule secreted by ectodermal cells of the developing hair follicle, appears to be critical in mediating the transition from telogen to anagen during postnatal hair cycling

184
Q
  1. Which of the following is true regarding BPAg1?

A. It is pathogenic in cicatricial pemphigoid
B. It is a member of the plakin family
C. It is pathogenic in pemphigoid gestationis
D. It is not pathogenic in paraneoplastic pemphigus
E. It coprecipitates with plakoglobin

A

Correct choice: B. It is a member of the plakin family

BPAg1 is a member of the plakin family, which includes envoplakin, periplakin, desmoplakin, plectin, and BPAg1. BPAg2 is pathogenic in cicatricial pemphigoid and pemphigoid gestationis.

BPAg1 is pathogenic in paraneoplastic pemphigus. Desmoglein 3 is pathogenic in pemphigus vulgaris and coprecipitates with
plakoglobin.

185
Q
  1. The first primordial hair follicles form on the eyebrows, upper lip and chin at which gestational age?

A. 7 weeks
B. 9 weeks
C. 12 weeks
D. 16 weeks
E. 20 weeks

A

Correct choice: B. 9 weeks

The first primordial hair follicles form at 9 weeks gestation on the eyebrows, upper lip and chin. The remaining follicles develop at 4-5 months in a cephalad to caudal direction. New follicles cannot develop in adult skin.

186
Q
  1. Apocrine glands:

A. Are coiled glands
B. Have a two segment ducts that empties onto the skin
C. Are present everywhere on the skin except on the palms and soles
D. Function from birth
E. Secretions are initially odorless

A

Correct choice: E. Secretions are initially odorless

Apocrine glands are tubular glands that demonstrate decapitation secretion. Like eccrine glands, the ducts are composed of three segments: intraepidermal duct, intradermal duct and secrectory portion. The duct usually leads to a pilosebaceous follicle above the entrance of the sebaceous duct. They are found in the axillae, anogenital region, external ear canal (ceruminous glands), in the eyelids (Moll�s glands) and in the breast (mammary glands). These glands are functional starting at

puberty. The initial secretions are odorless with the odor being derived from C6 � C11 acids. The most abundant being 3-methyl-2-hexenoic acid.

187
Q
  1. Epidermolysis bullosa simplex with muscular dystrophy is due to which defective molecule
    A. Plectin
    B. Keratin 5 and 14
    C. Laminin 5
    D. Collagen vii
    E. Integrin alpha 6 or beta 4
A

Correct choice: A. Plectin

EBS with muscular dystrophy is due to a mutation in plectin, plectin is also found in skeletal muscle. Keratin 5 and 14 are defective in EBS, EBS herpetiformis, EBS weber cockayne, EBS koebner. Laminin 5 is defective in Herlitz Junctional EB. Collagen VII is mutated in dominant dystrophic EB, Barts syndrome and EB acquisita. Integrin alpha 6/beta 4 is mutated in junctional epidermolysis bullosa with pyloric atresia. Epidermolysis bullosa (EB) is a group of inherited diseases that are characterised by blistering lesions on the skin and mucous membranes. These may occur anywhere on the body but most commonly appear at sites of friction and minor trauma such as the feet and hands. In some subtypes, blisters may also occur on internal organs, such as the oesophagus, stomach and respiratory tract, without any apparent friction.

EB should be distinguished from common friction blisters, and from epidermolysis bullosa acquisita (EBA), which is a blistering autoimmune disease that is not inherited and often doesn’t develop until adult life. The EB conditions result from genetic defects of molecules in the skin concerned with adhesion. Loss of adhesion results in blister formation. There are four major types of EB based on different sites of blister formation within the skin structure:

EB type The site of blister formation within skin
(EBS) Epidermolysis bullosa si Epidermis or uppermost layer of skin cells (keratinocytes)
(JEB)Junctional epidermolysis Lamina lucida within the basement membrane zone
(DEB)Dystrophic epidermolysi Lamina densa and upper dermis
Kindler syndrome Mixed pattern or multiple levels within and beneath the basement m
Within each of these types of EB, there are various subtypes. Varying degrees of severity that range from mild to severe are found with each EB type. International consensus on the diagnosis and

classification of EB has resulted in updated recommendations (2014), based on newer clinical and molecular data.

188
Q

188 -Sebaceous carcinoma stains positive for:

A. EMA
B. CEA
C. S100
D. CKIT
E. CD1A

A

Correct choice: A. EMA

Sebaceous carcinomas stain positive for EMA. The other answer choices are incorrect as they do not stain positive in sebaceous carcinoma.

189
Q
  1. The triple helix of the collagen molecule is largely maintained due to its amino acid composition. The polypeptide chains of collagen are repeating triplets of Glycine-X-Y.
    The X and Y positions can be occupied by multiple amino acids, but are most often:

A. Alanine and asparagine
B. Tyrosine and threonine
C. Histidine and ornithine
D. Leucine and isoleucine
E. Proline and hydroxyproline

A

Correct choice: E. Proline and hydroxyproline

Collagen fibers provide tensile strength to the skin and allow it to serve as a
protective organ against the external environment. Collagen makes up 80% of the dry weight of the dermis. The structure of collagen is comprised of three alpha chains arranged into a triple helix. The repeating amino acid triplets of the triple helix of collagen are most often Glycine-Proline- Hydroxyproline.

190
Q
  1. What percentage of the dry weight of skin in elastin?

A. 2
B. 4
C. 6
D. 8
E. 10

A

Correct choice: B. 4

Elastin fibers make up 4% of dry weight of the skin, forming a complex meshwork extending from lamina densa of the dermoepidermal junction through the dermis and into the hypodermis. Elastins help return the skin to the normal configuration after being stretched. Elastic fibers are 90% elastin wrapped by fibrillin microfibrils. Fibrillin is mutated in Marfan syndrome. The typical amino acids found in elastic fibers are desmosine and isodesmosine. Elastic fibers turn over slowly in the skin and are damaged by ultraviolet radiation.

191
Q
  1. Numerous neuromediators are involved in cutaneous neurobiology and many play a role in the development of inflammation in the skin. One such mediator can be induced by application of capsaicin to the skin. Which of the following is the correct neuromediator?

A. Noradernaline
B. Substance p
C. Neurokinin a
D. Acetylcholine
E. Pro-opiomelanocortin

A

Correct choice: B. Substance p

Substance P is a neuromediator that binds the tachykinin receptor. It is released upon stimulation of sensory nerve fibers. It has numerous cutaneous functions, including development of skin edema, erythema, and pruritus, upregulation of adhesion molecule expression, release of proinflammatory mediators, etc. Capsaicin causes release of substance P. This has been utilized pharmacologically in conditions such as zoster, in which consistent application of capsaicin leads to depletion of substance P, which in turn can help to reduce/eliminate post-herpetic neuralgia. The other answer

choices are also neuromediators that are involved in cutaneous inflammation, but their release is not induced by capsaicin.

192
Q
  1. Mast cells are derived from bone marrow + cells?
    A. CD3
    B. CD6
    C. CD20
    D. CD34
    E. CD68
A

Correct choice: D. CD34

CD34+ cells in the bone marrow are the precursors of mast cells. CD3 and CD20 are T and B cell markers respectively. CD6 is found on mononuclear phagocytic cells in the dermis, CD68 is a macrophage marker. Mast cells are normal cells in the body, usually found in the skin and other tissues. Mast cells have a role in the early steps of the body’s coordination of healing responses to an injury. Granules within the mast cells contain histamine and other chemicals.
* histamine
* leukotriene C4
* prostaglandin D2
* carboxypeptidase
* heparin
* cathespin G-like protease
* tryptase
* tumour necrosis factor-A
* chymase
* interleukin-8
* others

When a mast cell is activated, these chemicals are released into the surrounding skin. Mast cell chemicals are mediators of inflammation, and cause the blood vessels to leak, resulting in localised itching, swelling, redness and sometimes blistering. This is what happens normally in insect bites and is thought to be a protective mechanism. For example, a mosquito injects saliva when it bites. The saliva triggers mast cellactivation to a varying degree, depending on the individual’s hypersensitivity to the saliva. The unpleasant itch soon persuades the person to try to avoid getting bitten again.

Rubbing an area of skin affected by mastocytosis may also activate the mast cells. The rubbed skin becomes reddened, swollen and itchy within a few minutes (Darier sign). In young children, the rubbed area may later blister.

193
Q
  1. Dermal dendrocytes:

A. Are responsible for immediate-type hypersensitivity reactions.
B. Actively synthesize and release IgG.
C. Present antigen to naïve t cells in the lymph node.
D. Are the primary cell found in a glomangioma.
E. Are the primary cell found in an angiosarcoma.

A

Correct choice: C. Present antigen to naïve t cells in the lymph node.

A dermal dendrocyte is a mononuclear phagocytic cell that is a type of antigen presenting cell. It is derived from the bone marrow and found both in the papillary and upper reticular dermis. This cell is highly phagocytic and synonymous with the melanophage that has ingested pigment. Dermal dendrocytes are likely very important to the afferent limb of the immune response.

194
Q
  1. Which hair condition is associated with abnormal regulation of the peroxisome proliferator- activated receptor-à ŽÂ³ (PPARγ):

A. Alopecia areata
B. Lichen planopilaris
C. Telogen effluvium
D. Seborrheic dermatitis
E. Androgenic alopecia

A

Correct choice: B. Lichen planopilaris

Lichen planopilaris is a chronic, scarring condition that results in permanent loss of hair and destruction of the hair follicles. Defect in lipid metabolism and peroxisome biogenesis may contribute to LPP pathogenesis. Medications that act as PPAR agonists, such as pioglitazone, may

have a role in controlling the progression of this disease. (Karnik P, et al. (2009) Hair follicle stem cell-specific PPAR deletion causes scarring alopecia. J Invest Dermatol 129(5):1243-57)

195
Q
  1. Telogen effluvium:

A. Involves a diffuse alopecia affecting more than 50% of the scalp
B. Often results from antimetabolites used during cancer chemotherapy
C. Is an inflammatory alopecia devoid of scarring
D. Is a patchy alopecia affecting less than 50% of the scalp
E. Results from sudden illness or surgery

A

Correct choice: E. Results from sudden illness or surgery

Telogen effluvium is an excessive loss of club hairs from the normal resting follicles of the scalp. The follicle is pushed from anagen to catagen to telogen. There is no inflammatory process involved. Causes of telogen effluvium are illness, surgery, parturition, fever, drugs, traction, starvation, and hypervitaminosis A. Usually, the hair loss only involves less than 50% of the scalp. There is no specific therapy, and most cases are self-resolving within months.

196
Q
  1. The first cell type to migrate into a new wound in great numbers is the:

A. Neutrophil
B. Monocyte
C. Macrophage
D. Lymphocyte
E. Mast cell

A

Correct choice: A. Neutrophil

Neutrophils migrate with monocytes concurrently, but arrive first in great numbers because of their abundance in circulation.
Chemoattractants for the PMNs are fibrinogen, fibrin split products, C5a and leukotrienes. If wound contamination is controlled, PMN migration ceases within a few days and they become entrapped within the wound clot, undergo apoptosis or are phagocytosed by macrophages.

197
Q
  1. A 25-year-old man presents with a new diagnosis of plaque-type psoriasis. keratotic follicular papules involving the bilateral dorsal arms, thighs, and cheeks. A biopsy of an unaffected region of skin would likely exhibit which of the following findings:

A. Increased filaggrin
B. Increased loricrin
C. Increased involucrin
D. Diminished lamellar bodies
E. Decreased transglutaminase I activity

A

Correct choice: C. Increased involucrin

In psoriasis, involucrin levels are increased. In contrast, filaggrin and loricrin levels are diminished. Decreased lamellar bodies are implicated in Flegel’s disease andHarlequin icthyosis. Tissue transglutaminase I is faulty in lamellar icthyosis and non-bullous congenital icthyosiform erythroderma.

198
Q
  1. Itch is most commonly transmitted by:

A. C-polymodal nociceptor class nerves
B. A-delta class nerves
C. A-beta class nerves
D. Parasympathetic postganglionic fibers
E. A-beta and a-delta fibers

A

Correct choice: A. C-polymodal nociceptor class nerves

Itch is transmitted primarily by C-polymodal nociceptor class nerves. These are small diameter unmyelinated nerves that carry pain, thermal, mechanical and pruritic stimuli. A-delta fibers carry pain, thermal, mechanical and in some cases pruritic stimuli. A-beta fibers carry light touch and motion stimuli. Parasympathetic post-ganglionic fibers do not contribute to cutaneous pruritus

199
Q
  1. Which of the following enzymes does not require copper for functioning?

A. Lysyl oxidase
B. Atp7a
C. Tyrosinase
D. Cystathione beta-synthase
E. Ferrochelatase

A

Correct choice: E. Ferrochelatase

All of the listed enzymes are copper containing or dependent except ferrochelatase. Lysyl oxidase facilitates crosslinking of fibrillin in elastic fibers. ATP7a is deficient in Menkes Kinky Hair Syndrome. Cystathione beta-Synthase is defective in homocystinuria. Tyrosinase catalyzes the first 2 steps, and at least 1 subsequent step, in the conversion of tyrosine to melanin. Ferrochelatase mutation leads to excess protoporphyrin production and photosensitivity.

200
Q
  1. Which of the following statements about plectin is correct?

A. Links fillagrin to the plasma membrane
B. Crosslinks proteins in adherens junction
C. Mutations result in junctional epidermolysis bullosa with pyloric atresia
D. Plectin is a member of the plakin family
E. Plectin is a constituent protein of the desmosomal plaque

A

Correct choice: D. Plectin is a member of the plakin family

Plectin is a hemidesmosomal protein and is not present in the desmosome. It links intermediate filaments to the plasma membrane, crosslinks hemidesmosomal proteins, mutations result in epidermolysis bullosa simplex with muscular dystrophy and is a member of the plakin family.

201
Q
  1. Which of the following is not a specialized type of sebaceous gland?

A. Moll’s gland
B. Meibomian gland
C. Gland of zeis
D. Montgomery’s tubercle

E. Fordyce spot

A

Correct choice: A. Moll’s gland

There are several types of specialized sebaceous glands that are not associated with a hair follicle. They include Montgomery’s areolar tubercle, Fordyce spots of the lip, Glands of Zeis of the cutaneous eyelid, and Meibomian glands of the eyelid. Moll’s gland of the eyelid are a modified apocrine gland.

202
Q
  1. Glomus cells are composed of what?

A. Modified fibroblasts
B. Modified skeletal muscle cells
C. Vascular smooth muscle cells
D. Monocytes
E. Neuronal cells

A

Correct choice: C. Vascular smooth muscle cells

Glomus cells are derived from Suquet-Hoyer canals. They are vascular smooth muscle cells that allow the rapid shunting of blood from the arterioral to venularsystem, bypassing the capillaries. This process occurs primarily on the palms and soles.

203
Q
  1. Each of the following is true about melanosomes except:

A. Most characteristic organelle of the melanocyte
B. Tyrosinase activity decreases as melanosomoes mature
C. Are transferred to keratinocytes via phagocytosis
D. Are singly dispersed in the basal layer in white skin
E. Are larger in size in black skin compared to white skin

A

Correct choice: D. Are singly dispersed in the basal layer in white skin

Several differences exist that may explain the heterogeneity of skin color. The number of melanocytes are the same; however, there are several differences in the melanosomes. Differences in skin color can be attributed to five factors. In racially heavily pigmented skin, (1) there is greater production of melanosomes in melanocytes, (2) individual melanosomes show a higher degree of melanization, (3) melanosomes are larger, (4) the melanosomes are dispersed to a greater degree in the keratinocytes, and (5) there is a slower rate of degradation.

204
Q
  1. Which of the following cytokines are secreted from keratinocytes?

A. Il-2
B. Il-3
C. Il-17
D. Il-22
E. Il-23

A

Correct choice: E. Il-23

IL-2 is a cytokine that primarily produced by T lymphocytes and acts on
Other T lymphocytes as a growth factor for both survival anddifferentiation. IL-3 is also produced by T lymphocytes and acts as a growth factor for cells of the myeloid lineage. IL-17 is an effector cytokine that is implicated in the pathogenesis of psoriasis and produced by Th17 cells. IL-22 is also an effector cytokine produced by T lymphocytes that contributes to epidermal acanthosis in psoriasis. All of these cytokines are not secreted from epidermal keratinocytes. IL-23, which promotes Th17 effector function, is secreted by epidermal keratinocytes and upregulated in psoriasis. The p40 subunit of IL-23 is the target of drug ustekinimab used to treat psoriasis

205
Q
  1. Tissue contraction begins:

A. At 3rd day of wound healing
B. During the 2nd week of wound healing
C. After the first month of wound healing
D. After the 3rd month of wound healing
E. After the 9th month of wound healing

A

Correct choice: B. During the 2nd week of wound healing

Wound healing tends to be a predictable process that begins initially with the inflammatory stage. During this stage, clot formation occurs. This is the initial step in wound healing. Platelets, neutrophils and macrophages all migrate to the wound and secrete many mediators of wound healing. Epithelialization then begins hours after injury. This is followed by granulation tissue formation (four days after injury) and angiogenesis (first week of repair). Wound contraction ensues during the second week of healing.

206
Q
  1. The cutaneous immunofluorescence pattern in patients with Senear-Usher syndrome is:

A. Intercellular IgG and C3
B. Linear IgG and C3 along the basement membrane zone
C. Intercellular IgG on guinea pig esophagus
D. Intercellular IgG and C3 and linear IgG and C3 along the basement membrane zone

E. Linear IgG and C3 along the basement membrane zone and intercellular IgG on guinea pig esophagus

A

Correct choice: D. Intercellular IgG and C3 and linear IgG and C3 along the basement membrane zone

Senear-Usher syndrome, or pemphigus erythematosus, is a variant of pemphigus foliaceus characterized by crusted papules and plaques with a seborrheic distribution (malar region, scalp, upper chest and back). They demonstrate in-vivo IgG and C3 deposition on keratinocyte cell membranes and the basement membrane zone, in addition to circulating anti-nuclear antibodies.

207
Q
  1. S100 staining would be negative in?

A. Langerhans cells
B. Eccrine cells
C. Schwann cells
D. Adipocytes
E. Keratinocytes

A

Correct choice: E. Keratinocytes

S100 is sensitive for melanocytes and melanoma, but not specific as it also stains langerhans cells, eccrine cells, schwann cell, adipocytes, and chondrocytes. It is negative in keratinocytes and can help discriminate melanoma from pagetoid bowens or spindle cell squamous cell carcinoma.

208
Q
  1. Platelets release which of the following factors to promote new tissue growth?

A. Neutrophil chemotactic factor
B. IL-1
C. ADP
D. TGF-alpha
E. FGF

A

Correct choice: D. TGF-alpha

Platelets release PDGF, TGF-alpha and TGF-beta which promote new tissue growth. FGF and IL-1 are released from monocytes as growth factors (monocytes also release PDGF, TGF-alpha and TGF-beta). ADP is released from platelets, but is not functioning as a growth factor. Neutrophil chemotactic factor is released from mast cells and is an inflammatory mediator.

209
Q
  1. Which of the following domains is targeted by the autoantibodies in bullous pemphigoid?

A. NC16A of BP180
B. Laminin 5
C. Plectin
D. alpha-6 integrin
E. NC16A of BP230

A

Correct choice: A. NC16A of BP180

The NC16A domain of BP180 is the target of autoantibodies in bullous pemphigoid. It interacts with alpha-6 integrin extracellularly.

210
Q
  1. LEMD3 is mutated in which of the following syndromes?

A. Buschke Ollendorf
B. Albright Hereditary Osteodystrophy
C. Goltz
D. Mccune-Albright
E. Menkes

A

Correct choice: A. Buschke Ollendorf

LEMD3 or MAN1 is mutated in Buschke-Ollendorf syndrome. Common skin findings include dermatofibrofibrosis lenticularis disseminata. The radiographic finding commonly seen is osteopoikilosis. The gene defect was recently described as MAN1/LEMD3, a gene encoding for a nuclear membrane protein. Albright Hereditary Osteodystrophy is caused by inactivation of

GNAS1, Goltz is an x-linked dominant condition with unknown pathogenesis, McCune-Albright syndrome is caused by an activation of GNAS1. Menke’s Syndrome is caused by a defect in ATP7A/MKN on Xq13.

211
Q
  1. BP230 is a member of of which of the following families?

A. Plakin
B. Integrin
C. Laminin
D. Collagen
E. Elastin

A

Correct choice: A. Plakin

BP230 is a member of the plakin family and is homologous to desmoplakin and is intracellular. It attaches intermediate filaments to hemidesmosomal plaque. The remaining options are not part of the plakin family.

212
Q
  1. Which of the following dermal cells always express CD11c and CD6?

A. Mononuclear phagocytic cells
B. Fibroblasts
C. Mast cells
D. Erythrocytes
E. Glomus cells

A

Correct choice: A. Mononuclear phagocytic cells
The mononuclear phagocytic cell includes monocytes, macrophages and dermal dendrocytes. All phagocytic skin macrophages express CD11c and CD6. Fibroblasts do not produce a CD marker. Mast cells are derived from bone marrow-residing CD34+ stem cells. They do not produce CD11c or CD6.

213
Q
  1. Hair grows at:

A. 0.004 mm/day
B. 0.04 mm/day
C. 0.4 mm/day
D. 4 mm/day
E. 10 mm/day

A

Correct choice: C. 0.4 mm/day

Hair grows at an average of 0.4 mm/day. The other options are incorrect.

214
Q
  1. Which cytokeratin would you expect to be preferentially expressed in the keratinocytes of a psoriatic plaque:

A. K1
B. K5
C. K12
D. K15
E. K16

A

Correct choice: E. K16

K16 and K6 act as markers for hyperproliferative keratinocytes. They are found in skin disease such as psoriasis, warts, actinic keratoses, and SCC’s.

215
Q
  1. The following protein is the target for both dytrophic EB and Bullous SLE:

A. Type VII collagen
B. Laminin 5
C. Integrin subunit b4
D. Type XVII collagen
E. BPAG2

A

Correct choice: A. Type VII collagen

Type VII collagen is the target for both dystrophic EB and bullous SLE. It is also the target for EB acquisita.

216
Q
  1. Dystrophic epidermolysis bullosa results from mutations in:

A. Collagen type I
B. Collagen type II
C. Collagen type III
D. Collagen type V
E. Collagen type VII

A

Correct choice: E. Collagen type VII

Dystrophic epidermolysis bullosa results from mutations in Type 7 collagen. Type I collagen mutations are associated with osteogenesis imperfecta. Antibodies to Type II collagen are associated with relapsing polychondritis. Mutations in type V collagen result in the classic form of Ehlers Danlos while mutations in type III collagen result in the vascular form of Ehlers Danlos.

217
Q
  1. Krause end bulbs are:

A. Adapting mechanoreceptors found on weight-bearing sites that respond to vibrational stimul
B. Found on the vermillion border of the lips
C. Located in the dermal papillae of digital skin
D. Located in the deep dermis and within the subcutis in weight-bearing sites of the body
E. Found at the orifice of the hair follicle and particularly sensitive to cold

A

Correct choice: B. Found on the vermillion border of the lips

Krause end bulbs are mucocutaneous receptors found on the glans penis, clitoris, labia minora, perianal area, and vermillion border of the lips. Papillary nerve endings are free nerve endings found at the orifice of the hair follicle and particularly sensitive to cold. Meissner’s corpuscles are

ovoid, elongated mechanoreceptors located in the dermal papillae of digital skin that detect touch and light pressure. Pacinian (Vater- Pacini) corpuscles exist in the deep dermis and within the subcutis in weight-bearing sites and function as adapting mechanoreceptors that respond to vibrational stimuli.

218
Q
  1. Hair follicle development in the human embryo begins during:

A. 1st trimester
B. 2nd trimester
C. 3rd trimester
D. At the blastocoele stage
E. Within 2 weeks of fertilization

A

Correct choice: A. 1st trimester

Hair follicles begin in the third month of fetal life as a downgrowth of cells from the epidermis (3rd month=12th week=1st trimester).

219
Q
  1. Meibomian glands are:

A. Modified sebaceous glands
B. Found everywhere except on the palms and soles
C. Sebaceous lobules that feed into a lactiferous duct
D. Modified ceruminous glands
E. Modified apocrine glands

A

Correct choice: A. Modified sebaceous glands

Meibomian glands of the eyelids are modified sebaceous glands. Sebaceous glands are found everywhere on the skin except the palms and soles. Montgomery’s areolar tubercles consist of several sebaceous lobules feeding into a lactiferous duct. Ceruminous glands are apocrine glands of the external ear canal. Apocrine glands in the eyelids are Moll’s glands.

220
Q
  1. During embryogenesis, periderm cells of the fetus contain which of the following substances?

A. Ceramide
B. Glycogen
C. Free fatty acids
D. Porphyrins
E. Sebum

A

Correct choice: B. Glycogen

In week 7 of embryogenesis the surface ectoderm produces two layers. The external layer is the periderm which contains glycogen and gives rise to the stratum corneum by week 21. The other layer is the stratum germinativum.

221
Q
  1. 1 melanocyte has contact with:

A. 6 keratinocytes
B. 12 keratinocytes
C. 18 keratinocytes
D. 30 keratinocytes
E. 36 keratinocytes

A

Correct choice: E. 36 keratinocytes

One melanocyte and 36 keratinocytes make up one epidermal melanin unit. This melanocytes transfers pigment to these keratinocytes.

222
Q
  1. Which of the following statements about darkly pigmented races versus lighter pigmented races is correct?

A. The number of melanosomes in melanocytes are the same
B. The individual melanosomes have the same degree of melanization
C. The melanosomes are equal in size melanosomes
D. There are equal numbers of melanocytes

E. There is a faster rate of melanosome degradation

A

Correct choice: D. There are equal numbers of melanocytes

Individuals with darker pigmentation have an equal ratio of melanoyctes to
keratinocytes. Darker pigmentation is related to an increased number of melanosomes, increased melanization, greater size of melanosomes, and slower degradation.

223
Q
  1. All of the following are true regarding the formation of hair except:

A. The inner root sheath keratinizes by means of trichohyalin granules
B. Henle’s layer is outside of Huxley’s layer
C. The outer root sheath is a downward extension of the epidermis
D. Huxley’s layer contains melanin
E. The hair matrix becomes the hair and the inner root sheath

A

Correct choice: D. Huxley’s layer contains melanin

The inner root sheath is composed of three layers, which are the inner
root sheath cuticle, Huxley layer, Henle layer. None of these layers contain melanin. All three layer keratinize by trichohyaline granules and disintegrate when they reach the isthmus of the hair follicle.

224
Q
  1. The classical types of Ehlers-Danlos Syndromes (Type 1 and 2) lead to varying degrees of hyperextensible skin, easy bruising, wide, atrophic scars, and hypermobile joints. The underlying defect in this disorder is a mutation in:

A. Elastin
B. Fibrillin 1
C. ABCC6
D. Collagen V
E. Cystathionine beta-synthase

A

Correct choice: D. Collagen V

Classical types of Ehrlers-Danlos are manifested by varying degrees of
hyperextensible skin, easy bruising, wide, atrophic scars, and hypermobile joints. The genetic defect is in Collagen V. Elastin is mutated in Cutis Laxa. ABCC6 is defective in Pseudoxanthoma elasticum. Cystathionine beta-synthase is mutated in homocystinuria.

225
Q
  1. Which type of collagen in mutated in osteogenesis imperfecta?

A. Collagen I
B. Collagen II
C. Collagen III
D. Collagen IV
E. Collagen VII

A

Correct choice: A. Collagen I

Collagen type I is mutated in osteogenesis imperfecta. The COL1A2 gene is located on 7q22 and the COL1A1 is on 17q22. The other listed collagens are not involved in osteogenesis imperfecta. Collagen II is in cartilage and vitreous, Collagen III is in fetal skin, blood vessels and intestines, Collagen IV is in basement membranes and Collagen VII in anchoring fibrils and amnion.

226
Q

226- on electron microscopy, which cell demonstrates cytoplasmic projections and secretory granules?

A. Langerhans cell
B. Keratinocyte
C. Mast cell
D. Melanocyte
E. Macrophage

A

Correct choice: C. Mast cell

Electron microscopy of mast cells demonstrates large long villi at the periphery and round or oval secretory granules. Langerhans cells show a folded nucleus and rarely phagocytized melanosomes on electron microscopy. Their characteristic feature is the presence of Birbeck granules whose disk shape with one or two vesicles at either end represents a tennis raquet. Examination of melanocytes shows an absence of tonofilaments or desmosomes with characteristic melanosomes in various stages of formation. Macrophages often contain phagocytized material within phagosomes.

227
Q
  1. Which of the following is the most common genetic alteration seen in mucosal melanomas?

A. GNAQ
B. Cyclin Dependant Kinase 4/6
C. BRAF
D. KIT
E. MDM2

A

Correct choice: D. KIT

Mucosal melanomas tend to have an activating mutation in KIT. GNAQ mutations are seen in both uveal melanomas and blue nevi. Cyclin dependant kinase 4/6 binds cyclin D and together phosphorylate Retinoblastoma. MDM2 targets p53 for ubiquitination.

228
Q
  1. What is the major component of the anchoring fibril?

A. Type I Collagen
B. Type III Collagen
C. Type IV Collagen
D. Type VII Collagen
E. Laminin 5

A

Correct choice: D. Type VII Collagen

Anchoring fibrils are made of Type VII collagen. Anchoring plaques are made of Type IV collagen, and interact with a network of Type I And Type III collagen fibers in the dermis.

229
Q
  1. Which of the following markers are specific and reliable for Merkel cells?

A. CD20
B. HMB-45
C. CD34
D. CD3
E. CK20

A

Correct choice: E.Cytokeratin (CK) 20 is a reliable marker for Merkel cells.

CD20 is a marker for B-cells and CD3 is a T-cell marker. HMB-45 is used in staining for immature melanosomes and is reactive in melanoma. CD34 is used to stain dermatofibrosarcoma protuberans.

230
Q
  1. The epidermis is comprised of what type of cells?

A. Keratinocytes, Melanocytes, Merkel cells, Langerhan cells
B. Keratinocytes, Endothelial cells, Merkel cells, Langerhan cells
C. Keratinocytes, Melanocytes, Neutrophils, Langerhan cells
D. Keratinocytes, Melanocytes, Merkel cells, Goblet cells
E. Keratinocytes, Endothelial cells, Merkel cells, Goblet cells

A

Correct choice: A. Keratinocytes, Melanocytes, Merkel cells, Langerhan cells The adult epidermis is composed of three basic cell types: Keratinocytes,
melanocytes, and Langerhans cells. An additional cell, the Merkel cell, can be found in the basal layer of the palms and soles, the oral and genital mucosa, the nail bed, and the follicular infundibula.